Rawan 2022

Download as pdf or txt
Download as pdf or txt
You are on page 1of 174

‫تجميعات روان‬

‫شهر ‪9 + 10 + 11‬‬

‫آخر تعديل بتاريخ ‪11\11‬‬


First part:

Tyrosine residue in mass spectroscopy?

A- Leucine

B- Lysine

C- Serine

D- Phenylalanine✔

What to give a woman during labor who has strepto group B?

A- Ampicillin✔

B- Cefazoline

Counselling of warfarin?

A. Avoid green leaves

B.Take green leaves

C.Separate between green leaves and warfarin with at least 4 hours✔

D. Separate between green leaves and warfarin with at least 2 hour

give fixed amount of green leaves : ‫األصح‬

chewable tab?

A- can only chewable

B- chewable and swallowing✔

:‫واذا قال‬
:‫ يكون مفضل في‬: Chewable aspirin

Acute coronary syndrome

Warfarin used in pregnant women with (mechanical valve):

A- not used

B- first trimester
C- second trimester✔

D- third trimester

A ‫ نختار‬mechanical valve ‫لو ماقال‬

What is the cause of ergotism?

A.Candida Albicans

B.Aspergillus

C.Claviceps purpure

Case:

Edema + bacterial infection

‫عملو تزريع وطلعت مقاومه لواحد من مشتقات البنسلين م متذكرو‬


: ‫تتوقع البكتريا دي شنو‬

MRSA✔

MSSA

S.pyogens

‫ وبعاني من كسل سال من سبب الكسل شنو؟‬HTN ‫مريض عندو‬

Hydralazine

Isosorbide

Bisoprolol✔

Pregnancy in first trimester with HTN, avoided drug?

A- Hydrochlorothiazide

B- Ramipril✔

C- Labetalol

D- Amlodipine

Case with low k level and other electrolyte normal:


A- 20mmol kcl/100 NS polus

B- 90 mmol kcl/250 NS polus

C- 20 kcl/450 NS infusion for 15 minutes✔

D- 90 kcl/500 NS inf for 5 hours

C ‫ ساعات وفوق بس ان شاءهللا انها‬٤ ‫ الن تاخذ وقت من‬D ‫البعض قالو‬

Crhons disease + Fistula:

A- Salfalsalzine✔

B- Cipro

C- Prednisolone

A‫ هو االصح واذا مو موجود نختار‬infliximab ‫اذا موجود‬

‫ برضو صح‬Cipro+ Metro ‫واذا موجود‬

What is considered aerobic pathogen and can lead to epidemic?

A.Influenza virus

B.Rubella virus✔

C.korona virus

D.herps virus

Pt. remain in TPN 6month will cause : (CKD – Acid-base imbalance ✅– bone disease)

Woman come to the clinic with menstrual pain, and give

you 4 structures and asking about the most suitable one?

A.aspirin

B.ibuprofen

C.mefenamic acid

D.celecoxib

How does PEG increase half-life of preparations?


A. Decrease antigenicity

B.Increase solubility

C.Increase particle size

Patient diagnosed with HF with ejection fraction 35% what to give?

Spironolactone 12.5 m

To decrease mortality

Contraindication for Crcl 20

Spironolactone✔

Metolazone

Furosemide

Torsemide

-Ciprofloxacin major side effect?

A. Pale skin

B. Heartburn

C. Seizures✔

side effect lamotragine? Blurred vision✔

-Amitriptyline can cause:

A neuropathy

B/sedation✔

C/drowsiness|)

-Amitriptyline can treatment or side effect : neuropathy✔

Mother went to pharmacist her baby 1 month old


suffering from mild fever?

A. Paracetamol nasal metered dose 15 Mg/Kg

B. Ibuprofen syrup dose 12Mg/Kg

C. Naproxen

D. Digoxin

Osteomyelitis duration?

2-1 -weeks

6-4 -weeks

7 -days

Simvastatin max dose in geriatric?

A. 20Mg

B. 60Mg

C. 40Mg✔

D. 80Mg

Dose of heparin?

A. 80-100 units/Kg

B. 200 units/Kg

C. 10-100 units/Kg✔

D. 80-500 units/Kg

Maximum infusion rate of vancomycin?

A. 1 MI/Min

B. 2 MI/Min✔

C. 3 MI/Min

D. 4 MI/Min
Maximum dose of phyntoin?

A. 300mg/D

B. 400mg/D

C. 500mg/D

D. 600mg/D

Patient receive morphine 60mg every 12hr Dr will

change to fentanyl. What will be dose?

A. 2mg/D

B. 12Mg/D

C. 1.2Mg/D

D. 5Mg/D

Normal range of HbA1c is?

A. 4-7

B. 5-5.6

C. 4-5.6✔

D. 4-5

Pyrogen present in containers can be destroyed by

heating the containers at?

A. 121֯C for 30 Mins

B.210֯C for 1Hr

C.121֯C for 15 Mins

D.180֯C for 4Hr✔

Vancomycin the trough:

A- after 30 min 4 dose

B- after 30 min 3 dose✔


trough : Before 4 dose

Interleukin I inhibitor?

A. Digoxin

B. Anakinra

C. Methotrexate

D. Warfarin

Nicotine from tobacco is an alkaloid which is?

A.Crystalline

B.Oxygen free Solid

C.Semisolid

D.Oxygen free liqui✔

According to the Saudia food and drug authority

cough medications are contraindicated in children?

A. Less than one year

B. Less than two years

C. Less than three years

D. Less than six years✔

Patient with hot flashes and did hysterectomy

What for her

Estrogen✔

Estrogen + progesterone

A child is taking high dose amoxicillin (99mg/Kg) for Otitis media came to the clinic for pneumonia and
infuenza vaccine?

A. Cancel the vaccines


B. Delay the vaccines for one year

C. Proceed to give the vaccine✔

Skewed data which central tendency used?

Mean

Mode

Median✔

Standard deviations

Cause metabolic acidosis:

A- phenytoin

B-phenobarbital

C- valporic acid

D- topiramate✔

24-year-old pregnant female presents to the urgent care clinic with fever,

frequency, and urgency. She is diagnosed with a urinary tract infection (UTI).

Based on potential harm to the fetus, which of the following medications should be

avoided in treating her UTI?

A. Nitrofurantoin.

B. Amoxicillin.

C. Cephalexin.

D. Tobramycin✔ .

During prescription verification, a pharmacist identified a drug interaction in a 23-year-old woman


diagnosed with urinary tract and prescribed an antibiotic. Pharmacist contacted the change to a drug
that will not harm the patient.

Which of the following antibiotics is the best option for the patient?

A) Cephalexin✔
B) Ciprofloxacin

C) Nitrofurantoin

D) Trimethoprim-sulfamethoxazole

Which of the following medications can cause hemolytic anemia in the newborn when used

by a pregnant woman at full term?

A. amoxicillin

B. ciprofloxacin

C. azithromycin

D. nitrofurantoin✔

[Full term nitrofurantion]

1[trimester : ciprofloxacin]

58-year-old male with a history of hepatitis C, cirrhosis, and ascites

presents with spontaneous bacterial peritonitis. Which of the following antibiotics

requires close monitoring and dosing adjustment in this patient given his liver

disease?

A. Penicillin G.

B. Tobramycin.

C. Erythromycin✔.

D. Vancomycin

what is the recommended prophylactic therapy for pregnant women colonized with group B
streptococcus in labor with penicillin allergies at high risk for anaphylaxis?

A. Cefazoline

B. Linezolid

C. Penicillin G

D. Clindamycin✔.
A 28-year-old pregnant female in labor is admitted to the hospital. She tested positive for

Group B streptococcus colonization at her 36-week appointment. She has a penicillin allergy and her
culture are resistant to both clindamycin and erythromycin.

What is the recommended treatment for this patient?

A. Cefazolin

B. Penicillin G

C. Ceftriaxone

D. Vancomycin✔

36-Which of the following drugs is known to have significant variable

bioavailability due to its extensive hepatic metabolism upon oral administration?

A. Amoxicillin

B. Propranolol✔

C. Ciprofloxacin

D. Acetaminophen

64-year-old male presents with signs and symptoms of an acute gouty flare.

His doctor wishes to treat him accordingly to improve his symptoms. Which of the

following strategies would be the LEAST likely to acutely improve his gout

symptoms and pain?

A. Naproxen.

B. Colchicine.

C. Probenecid✔.

D. Prednisone

56-year-old female is discovered to have megaloblastic anemia. Her past

medical history is significant for alcoholism. Which of the following would be the

best treatment option for this patient?

A. Oral vitamin B12.


B. Parenteral vitamin B12.

C. Oral folate.

D. Oral vitamin B12 with oral folate✔ .

physostigmine vs pyridostigmine structure and ask about replace

pyridostigmine from the formulary

A) continue physostigmine b/c pyridostigmine can produce amphetamine

B) Replace to pyridostigmine b/c it can produce amphetamine

C) Replace b/c pyridostigmine has lower SE✔

D) Asa structure Cell can modify proteins by adding ubiquitin

fda ‫ اتعمل ليه تعديل واتضاف له حلقه بنزين في الطرف واتكون مركب تاني وانه هيكون‬rivastigmine ‫ سؤال تاني ان‬#
‫ لالستخدام في مرضى الزهايمر‬approved

‫انت رايك ايه كصيدلي‬

A. Avoid because will produce amphetamine✔

B. Use it because it will produce amphetamine and be dual benefit

C. Use alternative more potent

Which is stimulatory which is inhibitory!?

-platelet activation factor

-Slicing✔

less sedative antipsychotic??

Quotation

Redispersion✔

Which has ceiling effect?

Morphine

Ibuprofen✔
Fentanyl

Oxycodone

Decrease the amount of minimum alveolar concentration (MAC) for anesthetic drug?

Diazepam✔

Patient with G6PD and has UTI?

Meropenem

Moxifloxacin

levofloxacin✔

Azithromycin

‫ سنين؟‬5 ‫ كل سنة وال كل‬DM1‫اختبار النظر لل‬

‫ سنين‬٥ ‫ كل‬Dm1

‫ كل سنه‬DM2

‫ ؟‬cough‫ بتقلل ال‬dry cough ‫اي نبته لو اخدها واحد عندوا‬

clove oil

eucalyptus oil✔

‫سؤال اول مانجمنت لمريض الضغط ل‬adults:

Aliskiren

Nifedipine✔

Thiazide

Bb

: ‫في شيماء قراته‬


Nifedipine for adults

Amlodipine for elderly


‫‪-‬سؤال يقول في ‪ tpn‬ايش الدوا الي نحسب له ال ‪ calories‬في الخيارات ‪ propofol‬و ‪ tigecycline‬؟‬

‫✔‪propofol‬الصح‬

‫‪ ,‬ايش اللي يتغير؟‪solutions‬التحضيرات اللي نسويها من تابلت الي‬

‫‪Side effects‬‬

‫‪Effect of drug‬‬

‫✔‪Stability of drug‬‬

‫✔‪-Hydrolysis‬يقول اذا تركت علبه االسبرين بالحمام ايش راح يصير لها ؟‬

‫‪ Influenza vaccine-‬يستخدم لألشخاص من عمر‬

‫‪A-All from 2 years‬‬

‫‪B-With comorbidity‬‬

‫✔‪C-All from 6 month‬‬

‫وكيس مريض عندو االم في البطن والطبيب فحصو بوضع اصبعو على مكان االلم ايش نوع الفحص ؟‬

‫✔‪Percussion‬‬

‫‪ every 12 hours‬ويأخذ ڤانكومايسن‪MRSA ،‬فيه كيس بعد شخص عنده‬

‫ولما قاسوا الليڤل حقه صار ‪ ،٥‬والخيارات ‪:‬‬


‫‪-Stop vancomycin and start linezolid‬‬

‫✔‪-Increase the frequency of vancomycin to every 8 hours‬‬

‫الممكن نستخدمها مع المريض‪ dvt‬جا يستشير الصيدلي عن مصدر لمعلومات عن ال‪colorectal‬جراح‬

‫ما متذكرتهم االسماء طويلة بس مضمونهم‬


‫‪A-Dvt with orthopedic‬‬

‫✔‪B-Dvt non orthopedic‬‬

‫‪C-Parental nutrition and interna‬‬


Lacoste ‫ وحطو فيه‬E.coli ‫في طبق زرعوا بكتيريا‬

‫ايش رايك يصير‬


A- Damage

B- Nutrition✔

‫صوت صمامات القلب‬dup lub:

A- S1,S2 .mitral valve✔

B- S1, S2, S3 mitral valve

What is the function of pili in bacteria?

A. bacterial movement

B. attach a bacterial cell to specific surfaces✔

C. key virulence characteristic

D. all of the above‫اذا موجوده‬

Which enzyme inhibit bacterial cell wall?

Lysosoma

Lysosome✔

Pregent women with otitis media give?

Cipro

Levofloxacin

Moxifloxacin

Azithromycin✔

7 -drug can cause iron deficiency?

A- Magnesium hydroxide

B- Levothyroxine

C- Metformin✔
Laxative slow onset of action?

A- psyllium

B- glycerin

C- senna

D- lactoluse✔

A ‫لو غير موجود نختار‬

‫ايش من‬phase ‫ يكون‬pharmacovigilance

Phase 1

Phase 2

Phase 3

Phase 4✔

‫يقول‬potentiate of oral hypoglycemia:

Digitalis

Green tea

Chamomile✔

Coenzyme Q10

‫ من ايش؟‬toxicity ‫واحد راح المستشفى مصاب بحرق وحصل‬-

Cyanide✔

Lead

Arsenic

Q38: Inherited genetic ..location..1-22 and x/y?

A. Autosomal✔

B. Heterozygous allele

C. Aneuploid
D. Acrocentric

Q39: Chromosomes 13, 14, 15, 21, and 22 are?

A. Acrocentric✔

B. Heterozygous allele

C. Aneuploid

Straight rods shape

A) bacilli✔

B) cocci

Bacterial flora in skin?

Streptococcus epidermis✔

Have cell membrane?

Bacteria✔

virus

Drug used for horizontal laminar flow?

TPN✔

Drug mask hypoglycemia? Beta blocker✔

Medication used with exercise induce asthma? A- Tiotropium

8 -Cromolyn

C- Cortisone

D- Salmeterol✔

Which of the following is not secondary cause of dyslipidemia..

A.hypothyroidism
B. hyperthyroidism✔

C . diabetes

D. protease inhabited

-Anesthetic with higher chances of cardiotoxicity: nitrous oxide✔

-Anesthetic with higher chances of hepatic toxicity : isoflurane✔

Main features of graves’ disease include:

A. diffuse goiter

B. thyroid eye disease✔

C. pretibial myxoedema

D. thyroid acropathy

Patient female with food poisoning and went to hospital with troponin high NSTMI what is the
management?

A- aspirin 81 mg + ticagrelor✔

B- Aspirin 325 mg + ticagrelor

C- Aspirin 81 mg +Prasugrel

D- Aspirin 325 mg + prasugrel

women take levothyroxcen and She is planning to get pregnant, ask pharmacist about herb an
alternative?

-Tell her that Levothyroxine is safe during pregnancy✔

Which of the following perspectives incorporates all the costs and regardless of who incurs or obtains
them?

A) Patient

B) Hospital

C) Societal✔

D) Ministry of Health
Patient got severe head injury due to accident and went to primary and taken to secondary and 2dry
refer him to tertiary. What is this?

A) Primary to secondary

B) Secondary to tertiary✔

C)Primary to specialist

d)secondary to specialist

The pH of a buffer system can be calculated with the

(A) Noyes–Whitney equation.

(B) Henderson–Hasselbalch equation✔

(C) Michaelis–Menten equation.

(D) Stokes equation

Acute Heart failure Patient with shortness of breath (SOB) and peripheral edema,

which forester classification?

A- dry and cold

B- wet and cold✔

C- dry and warm

D- wet and warm

Horizontal laminar flow:

A- Cefepime✔

B- Anti cancer

ventricle laminar flow:

Anti-cancer ✔ or hazard medicine

Warfarin what is the type of bond? Conjugation✔


Which is volatile alkaloids?

A- Nicotine✔

B- Theophylline

Drug cause bradykinin and hypersalivation?

Digitalis

ACE inhibitor✔

Digoxin

UFH cause:

Hypokalemia

Hyperkalemia✔

-Isoniazid monitoring: Hepatic✔

-Methotrexate monitoring:

Cbc , Renal and hepatic✔

-Cyclosporine monitoring trough level?

Before 4th dose✔

Epileptic patient has skin rash after use of oxcarbazepine what is alternative therapy?

Ethosuximide✔

Vaccine and dental work what type of demand:

A. Postive✔

B. Negative
Relief pain dysmenorrhea,

Heat water✔

Cold water

Reduce exercise

-The gene HER2 was negative we can use:

Letrozole✔

-The gene HER2 was positive we can use:

Trustzomab✔

Vaccine give long life immunity?

A-Varicella

)‫ نختارها اذا ما في‬bcg( ✔ B- Measles

C- Pcv

D- Mcv

BCG ‫ الصح‬،‫هنا الخيارات كلها غير صحيحه‬

Which of the following vaccines is usually

not given until after the child's first

birthday?

a. Influenza vaccine

b. MMR vaccine✔

c. DTap vaccine

d. Hib vaccine

How to measuring pain in child?

1 -numerical rating scale

2 -verbal rating scale


3 -visual analog scales

4 -face pain scale -revised✔

contraceptive with: breast feeding:

.A-Estrogen pill

B-. Progestin pill✔

C-monophasic.

D - Diphasic

Which is teratogenic especially in third trimester:

a) Ibuprofen✔

b) Lithium

c) Methoxrate

Which of the following statements describes unfractionated heparin?

A. enolic acid derivative

B. propionic acid derivative

acetyl ester of salicylic acid

D. mucopolysaccharide polymers✔

Lipophilic drug excreted in breast milk?

Diazepam✔

Dihydroergotamine

Which type of anemia can be treated by chemotherapy anticancer?

A/Non lymphocytic anemia

B/Acute lymphoid anemia✔

C/Aplastic anemia

D/Hemolytic anemia
Each individual involved in manufacturing of sterile

preparations is required to be well trained to produce sterile preparations, which is the following tests is
required by USP to be done annually to ensure consistent

production of sterile drugs?

A. Sterility test

B. Media fill✔

C. Viable and non-viable environmental monitoring

primary information resources?

-GUIDELINES FROM CDC

-RCT FROM PubMed

-handbook information Review article✔

Which of the following is not commonly considered to be part of the public health function of
community pharmacists?

a) Health promotion

b) Compounding✔

c) Diabetes testing

d) 'Sign-posting'

What is the acceptable bioavailability in self life?

98%

97%

‫ دكتور جاوبها‬90%

‫ ✔ انا اشوف الصحيحة‬87%

Otic condition treated without prescription agent?

A. Vertigo
B. Water clogged ear✔

C. Tinnitus

D. Temporary hear loss

Sphygmomanometer cuff placed around which part? Arm✔

To reach glucagon in muscle in diabetic patient he needs to

Moderate exercise

Mild Exercise

Hard exercise

Relax muscles✔

Which of the following counseling points is most important to discuss with a

patient who is taking carbidopa/levodopa?

A. Should be taken on an empty stomach

B. High-fat meal may increase the absorption

C. Should not be taken with calcium supplements

D. High-protein meal may decrease the absorption✔

Patient with infection, hypotension, and high HR. What to give

a- hypertonic saline✔

b- 0.9 NS

c- dextran

d- ringer lactate

Treatment of metabolic alkalosis?

Ca carbonate

Saline.9✔

Ringer lactate. (CI)


Dopamine iv

Question about tablet punching:

A. chipping,

B. capping✔ ,

C. binding

powder stays in bottom?

Chipping

Cling✔

Capping

45-year-old woman was prescribed a sedative for anxiety to be taken at night. Three days later, she
presented complaining of drowsiness, dysphoria and motor depression every morning.

Which of the following medications was most likely prescribed?

A. zolpidem diazepam

B. ramelteon✔

D. midazolam

9-year-old boy had nail piercing few days ago, now he has osteomyelitis, which organism?

A. E.coli

B. pseudomonas aeruginosa

C. Streptococcus pyrogens✔

D. Candida albicans

‫لو فيه‬Staphylococcus‫نختارها‬

important of using patches as contraceptive?

A. To remove compliance

B. For long duration of action‫✔✔ األصح‬


C. To prevent first pass metabolism✔

‫اذا فيه خيار يتعلق بالحرارة نختاره‬

20kg take illicit drug Antidote‫؟‬

Naloxon 400 oral

Naloxone 400 im✔

Flumazenil oral

Flumazenil im

Child drink bleach what to do?

Give acid

Give alkaline✔

Evacuations

♀ ‫ قالو نختاره‬supportive care ‫اذا فيه خيار‬

What are the therapeutic outcomes for a patient with type 2 diabetes mellitus?

a) To keep blood glucose levels above 4 millimole per litre

b) To keep blood glucose between 4 and 9 millimole per litre

c) To prevent hypoglycaemia

d) To prevent hyperglycaemia✔

Which vitamin affects erythropoietin?

A- Iron deficiency

B- Thiamin deficiency

C- Iodine deficiency

D- B12 deficiency✔

A ‫اتوقع لو ماطلب الفيتامين بالسؤال تكون االجابه‬

156●-Reaction (NAD _ NADH+) give + 30 energy?


1 .exogenic-

2.endogenic✔ -

3.Endothermic

.4 -exothermic

Face to face communication barrier??

A. No elaboration or discussion

B. Disturbs the clinic and patient flow✔

C. Delay response

Patient taking garlic with warfarin and the pharmacist want to know side effect?

A. Natural product review✔

B. Micromedex

Oral bioequivalent dose of IV furosemide 40 mg when its bioavailability is 65%? ‫حاجة كدة‬

20

40

60

80✔

The lup-dup sound results from?

A. S1, S2, S3, and S4 mitral, tricuspid, aortic, and pulmonary valves closure

B. S3, S4...

C. S1, S2, S3 mitral, tricuspid, aortic, and pulmonary valves closure✔

D. S1, S2, S4 murmur...

Patient had car accident and was taken to the nearest community hospital where he had

surgery and required casting for 8 weeks, after that he needs rehabilitation and he was

??referred
.A. Tertiary to rehabilitative✔

B. Secondary to rehabilitative

patient came to satellite pharmacy in ER and says his is out of insulin, what you should do?

A. Tell him to buy from retail pharmacy

B. give him one vial

C. tell him you cant give him without prescription✔

D. tell him to go to ER to assess his case

:The degradation reaction of ASA involves

a.hydrolysis✔

b.racemization

c.oxidation

d.photolys

Digoxin toxicity

Patient with hypokalemia and hyponatremia. The nurse was able to open only one line:

a- K 15 mmol in 250 ml NS same line✔

b- K 15 mmol in 250 ml NS different line

c- Na 15 mmol in 250 ml NS same line

Which found in mitochondria

coenzyme q with iron.

coenzyme will with selenium✔.

prophylin with iron.3

prophylin with selenium.

What to give for patient take metformin?

Vit B12✔
Drugs can precipitate gout:

1 .Diuretic✔

2 Aspirin

3.Niacin

4.levodopa

5.cyclosporine

Master budget?

Cash✔

Sale

Operation

Expensive

QT represent?

Atrioventricular

Ventricular depolarization

Plateau

Ventricular action potential✔

Value Chain Service Providers offer:

a) Software collaboration between partners

b) Information flows across the value chain

c) Specialist services or functions within the value chain✔

d) Information brokerage across a sector

Observing evidence of instability during compounding is an important step.

Which of the following is happened should have discarded the product and not

dispense it?
A. Color intensity✔

B. Weight Changing

C. Size of the holder

D. Chemical potency

What is the normal immunological role of the CD8+ve T-cell

a) Helps B-lymphocytes to develop into plasma cells.

b) Kills virus infected cells✔.

c) Secretes antibodies

d) Rejects transplanted tissue.

9 40 -years old women with breast cancer, topoisomerase is elevated, which drug should

be used

A- Cyclophosphamide

B- Melphalan

C- Irinotecan✔

D- Methotrexate

Drug of choice for patient has focal seizure with HLA-B 1502 positive?

A. levetiracetam✔

B. Carbamazepine

C. Lamotrigine

D. Phenobarbital

pharmacist showed a patient how to use injection by pictures?

A. Verbal

B. Nonverbal

C. Physical

D. Visual✔
Ampicillin weak acid ..what's fraction will be in lipid form when ampicillin in duodenum (PH 4)??

1%✔

99%

50%

80%

7-mediators in the mucosa?

IgG✔

-IgE

-T-cell

-Macrophages

‫ نختارها ألنها األصح‬IgA ‫اذا موجوده‬

Decrease or increase the dose by technician is:

A) Omission error

B) Prescription error

C) Dispensing error✔

D) Personal error

-A nurse ordered epinephrine from the pharmacy, 15 minutes later she called the

pharmacy and no one answered. Then she took a vial form the crash cart and prepared it

for the patient herself and administered safely to the patient. What did this nurse do

wrong?

A. prepare the medication herself

B. Used the crash cart for non-emergency cases✔

C. There was a high chance of error

Anticoagulant take with meal?

Rivaroxaban✔
Argatroban

Pertussis management for unvaccinated 1year baby (profen - azithromycin-✔

amoxicillin - dexamethasone)

Patient came with INR 11 and minor bleeding what to do

‫ ✔✔ األصح‬A/ oral Vit K

B/ IM Vit K

C/ Prothrombin complex concentrate

D/ Recombinant activated factor VII

:‫*للتوضيح‬
oral vit k :‫ نعطيه‬8 ‫ تحت‬INR ‫ لو‬minor bleeding

.)‫ (ولكن غير موجوده‬iv vit k :‫ نعطيه‬8 ‫ فوق‬INR ‫أما لو‬

iv : ‫ نعطيه‬coagulopathy ‫) واذا معه‬neonate ‫ (بحالة ال‬im vit K ‫*ال‬

-Patient has mild pain of osteoarthritis

Indomethacin

Ibuprofen

Paracetamol✔

Diclofenac

A patient was administered a neuromuscular blocker (NMB) prior to a surgical procedure to produce
skeletal muscle paralysis. This NMB drug affected small, rapidly contracting muscles of the face and eyes
first and diaphragm muscles last. The effect of this drug was easily reversed with neostigmine. Which of
the following neuromuscular blockers was most likely administered to this patient?

A. Rocuronium.

B. Succinylcholine✔ .

C. Diazepam.

D. Tubocurarine.
Which of the following could lead to decrease in international normalization ratio (INR) in a

patient of warfarin therapy?

A. decrease the consumption of dietary vitamin K

B. carbamazepine therapy✔

C. cotrimoxazole therapy

D. metronidazole therapy

Glucosamine for uses??

A- Osteoporosis

B- Osteomyelitis

C- Osteoarthritis✔

-Structures of zidovudine antivirals, marked 4 area on structure and asked about

which part to remove that make the antiviral drug not effective?

A. N3✔

B. CH3

C. C=O

D. O

]antiviral effect. OH[*

]termination N3[*

Lithium and Gentamicin can be measured and curved

down after?

A. 8 Hours✔

B. 7 Hours

C. 5 Hours

lithium dose should be withdrawn after dose by?

6 hr – 8 hr – 12 hr✔
Which of the following is the mode of action of Phentolamine?

A. beta-adrenergic blocker

B. beta-adrenergic agonist

C. alpha-adrenergic agonist

D. alpha-adrenergic bloc✔

-Drug acid Where will be high execrations?

A. Pka 3.3 weak base

B. Pk 3.3 weak acid

C. Pka 6.8 weak base✔

D. Pka 6.8 weak acid

Progressive of disease?

1 -prevalence

2 -incidence✔

What the pulmonary HTN??

A. 10-20

B. 30-50

C. 25-30✔

D. 5-10

Which of the following proteins is a protease of the blood clotting pathway

a) Factor XII‫✔✔اتوقع ذي األصح‬

b) Prothrombin

c) Fibrin

d) Plasminogen✔

‫ اختلفو فيه‬A ‫وبرضو هنا االغلب قالو ان االجابه‬


Pt with ischemic stroke when discharge from hospital must refer for which unit

(cardio – psychiatric – occupational ✔- neuroradiologist)

206-Head of pharmacy (decide) to make group to arrange formulary drugs in hospital?

A. organizing

B. planning✔

C. Leading

b ‫ و‬A ‫اختلفوا فيها ما بين‬

Child one years ago, had immunodeficiency, he took before 4-month IV Ig, and now it’s time to take
MMR Vaccine?

A- Give vaccine

B- don’t give because he took IVIG

C- don’t give because he has immunodeficiency✔

99)Patient come with allergic signs after made serological test it was show high titre of IgM and IgG

both what does it mean??

A. Past infection with same MO

B. New infection.

C. Past vaccine and now get real infection✔

)512-What to test prior using of methotrexate?? Liver✔

)513-What to administer prior using of methotrexate?? Folic acid✔

NSAIDs has the longest duration of action?

Piroxicam✔

Ibuprofen

Naproxen
Diclofenac

Milk pasteurization:

At 63 c temp for 30 min✔

A 52-year-old woman is known to have alcoholic liver disease.

Which of the following changes to pharmacokinetics and pharmacodynamics may be important to


consider when prescribing for her?

C. Reduced bioavailability of lipid soluble drugs

D. Reduced hepatic enzyme action✔

HAM need to ECG monitor?

A) Digoxin tab✔

B) Insulin

C)Kcl infusion‫✔✔ األصح‬

D)Norm saline

which more OTC poisoning

Infant✔

Child

Adult

Elderly

Patient went for hiking after that with 48hrs he started to get rash fever what type of

immune response ?"

Type 1

Type 2

Type 3

Type 4✔
27 )how is the nomenclature of these plants obtained ( Atropine belladonna and piper negra?)

-Ac, color✔

-Geography, color

-Discover, Ac

‫ للتوضيح‬Ac: active constituent

‫االتروبين عشان االكتف و البيبر عشان اللون‬

Which Amino acid we give for anorexia?

A. Alanine

B. arginine

C. valine✔

‫ تختاروها هي األصح‬leucine ‫لو فيه‬

Nurse received extra vial in the bag what should do?

A. Keep until next dose

B. Return to pharmacy✔

Dose related Ses of 200mM/L phenytoin?

A. Coma

B. Nystagmus✔

C. Ataxia

D. Decrease Mental status

What is the cell that produce monoclonal antibodies?

A.T cells

B.B cells

C.Meyloma

D.Hybridoma✔
What should be monitored at base line of TPN?

A.Glucose in blood

B.Sodium in serum

C.Triglycerides in serum

D.Potassium in seru✔

Diabetic patient on metformin, pioglitazone and sitagliptin.) Screening shows positive hepatitis C and
elevated liver enzymes. What is the most appropriate action?

A-Do not change

B- Stop pioglitazone

TZD antidiabetics class are hepatotoxic

stop pioglitazone✔

C- Stop sitagliptin

D- stop pioglitazone and sitagliptin

Which of the following is a function of alcoholdehydrogenase

enzyme?

•Accelerates the reduction of ethanol

•Accelerates the addition of hydrogen to ethanol

•Accelerates the addition of phosphate to ethanol

•Accelerates the removal of hydrogen from ethanol✔

183 -Long term plan use all aspects in organization

A- strategy

B- operation

C- organization
184 -pharmacist wrote email to another pharmacist to ask add medication to the formulary in the
hospital this is consider as?

A. Inter institutional

B. Intra institutional

C. Written communication

D. Non Verbal communication

pharmacist manager screemed on one of the pharmacist who dispensed wrong

medication in front of all the staff what is the good response?

A. he should punish him in separate area

B. ask for a meeting with the staff

C. cut from his salary

D. try to find the cause of the error and set solutions

Which of the following is the function of the pharmacy and therapeutic committee in a hospital?

A. Responsible for poison management

B. Monitor the adverse reaction reporting

C. Monitor the medication error reporting

D. Responsible for drug formulary management

A 65-year-old man presents to the pharmacy for a refill his medications. On review of his refill history,
the pharmacist noticed that he does not refill his medications regularly. Upon asking him, the patient
started that he has multiple medications that require complex administration.

Which of the following non adherence factors affecting above patient?

A. Patient related factors

B. Provider related factors

C. Health care system factors

D. Medication and condition factor


It is the broad range of health care services provided by in order to optimize therapeutic outcomes for
individual patients

Which of the following is the term used to describe the above- mentioned definition?

A. Pharmaceutical care

B. Medication counseling

C. Medication reconciliation

D. Medication therapy management

drug is used to treat an anaphylactic reaction?

A. Atropine

B. Adrenaline

C. Isoprinosine

D. Phentolamine

Which of the following is a property of amphetamine?

A. It does not cross the blood brain barrier

B. It stimulates the release of stored acetylcholine

C. It stimulates the release of stored norepinephrine

D. It stimulates the release of five hydroxytryptophan (5-

Endotoxin part of gram?

negative✔

positive

which is cytokine?

A. Leukotriene✔

B. Adrenaline

C. Gonadotropin
55-year-old man presents with an excessive bronchial and oronasal secretions. History

revealed, is exposed to an organophosphate insecticide. Which of the following is the best

management?

A. Atropine IV✔

B. Diphenhydramine orally

C. Pralidoxime intramuscularly

D. Physostigmine intramuscular

First line treatment COPD:

A/Inhaled corticosteroid

B/Bronchodilators✔

Women suffer from acute asthma attack what is the best choice for her attack?

A.ipratropium

B.tiotropium

C.corticosteroids✔

Status asthmaticus?

1.ipratropium

2 .Corticosteroids✔

Cast about status asthmaticus?

SABA✔

LABA

Ics

Methylprednisolone

Which of the following Clostridium spp. Is linked to the develop pseudomembranous

colitis?
A) C. tetani

B) C. difficile✔

C) C. botulinum

D) C. Perfringens

Naproxen adverse affects?

1 -cirrhosis

2 -renal failure✔

3 -hypotension

4 -irritable bowel syndrome

16-month-old baby with bronchopulmonary dysplasia is admitted Pediatric Intensive Care

Unit with respiratory distress requiring in (see lab results)

HEART RATE 86/min

RESPIRATORY RATE 30/min

TEMP. 38.6°C

Nasal swab Positive for respiratory syncytial virus, What is the best management?

A) Cefuroxime

B) Palivizumab

C) Corticosteroid

D) Intravenous fluids and supportive care✔

An 18-year-old healthy girl admitted to the hospital with a diagnosis of necrotizing

pneumonia. She received vancomycin 1 gm IV q12h and meropenem 1 gm IV q8h. The trough level of
vancomycin was 16 mcg/ml (normal range: 10- 20 mcg/ml).

Which of the following is the next appropriate step in management of this patient?

A. Continue the same treatment✔

B. Request a drug level for meropenem

C. Increase the dose of vancomycin to 2 gm IV q12h


D. Decrease the dose of vancomycin to 500 mg IV q12h

Which of the following patients are most at risk of suffering from an adverse drug reaction?

a) An 8 month year old infant receiving a prescription for an antibiotic.

b) A 22 year old patient with asthma receiving prescriptions for inhalers to relieve and prevent their
asthma.

c) A 48 year old patient who has hypertension and receives a prescription for an ACE inhibitor.

d) A 68 year old patient who has oedema receiving a prescription for a diuretic✔ .

lactating mother with asthma which can affect milk production?

A-steroids✔

B-LABA

C-LABA

D-SABA

Patient with arthritis wants to start on

methotrexate he is 65 years old. When he should take the vaccination??

A. Immediately.

B. Wait until finished from methotrexate course of therapy✔.

C. Give the vaccine after 2 month start methotrexate.

A 2 years old child came to clinic for taking hepatitis A vaccine, we know

that he took pneumonia vaccine from month ago, so we should:

a) Give him the vaccine immediately✔

b) Wait for 3 months

c) Wait for 6 months

d) Wait for 1 year

patient has hypertension and Benign prostatic hyperplasia


take Lisinopril, prazosin another antihypertensive drug but still

uncontrol He is have side effect Orthostatic hypotension

A-Add finasteride

B- Stop prazosin

C- Discontinue prazosin and change to Alfuzosin✔

Old patient has hypertension and multiple medical condition, have recently BPH ،he took lisinopril
and oxybutynin , what should the doctor do:

Start prazosin

start alfuzosin✔

continue oxybutynin

discontinue oxybutynin

patient with angina and MI who takes ACEI and many other medicines. What

should we give for high blood pressure?

A. spironolactone

B. carvedilol✔

C. diltiazem

D. ARBs

If patient on warfarin therapy decide to be on vegan diet what will be the effect on INR?

Increase

Decrease✔

Not effected

Not related

-Ibuprofen interaction with warfarin?

A)Ibuprofen increase metabolism of warfarin

B)Ibuprofen decrease metabolism of warfarin


C)Ibuprofen displace warfarin from protein binding✔

What anticoagulant needs bridging by Iv route before oral dose?

A. warfarin

B. Rivaroxaban

C. apixaban

D. dabigatran✔

First line anticoagulant for metral replacement?

Warfarin✔

Colpidogerl

Dapigatran

Enoxaparin

Interaction between warfarin and TCA as amitriptyline?

1 .increase bleeding risk✔

2 .increase adverse effects of warfarin

3 .decrease warfarin metabolism

4.increase please level

which increase the risk of kidney failure?

a) age

b) diabetes✔

c) hypertension

d) radioactive iodine

6 -which anticoagulants used after food

A/warfarin

B/apixaban
C/ rivaroxaban✔

D/ dabigatran

1-Patient was on olanzapine and not well controlled random glucose test was high, what is the best
alternative?

A) aripiprazole✔

B) clozapine

C) donepezil

D) haloperidol (1 generation)

2 -patient diagnosed with Provoked DVT his weight 70 and hight 170 and age 40 and CrCL 40, What is
the treatment for him?

A) warfrin

B) Apixaban

C) enoxaparin 40mg OD

D) Enoxaparin 70mg BID✔

The recommended DVT prophylaxis dose for postpartum in women with heigh risk of DVT is?

A. 3 weeks

‫محمد سليمان والهواري اتفقو عليها‬.‫✔ د‬B. 6 weeks

C. 3 months

D. 6 months

:‫للتوضيح‬
*Postpartum:6 week*

*Unprovoked DVT: 6 month*

*Provoked DVT: 3 month*

2 patients taking warfarin with different metabolism the reason may be due to?
A/Pharmacodynamics

B/phramcogenomics✔

Warfarin with levofloxacin

A/INR dec

B/ INR inc✔

Affect off Q10 with warfarin Therapy

A/ increase

B/ decrease✔

Pt use Warfarin 10mg and INR is low why?

A. Low dose

B. Diet✔

C. Low bioavailability of war

Who can open pharmacy in saudia‫؟‬

A- Only Saudi citizen✔

B- Can be non Saudi but register pharmacists should be there to manage shifts

C- Can be non Saudi but manager should be Saudi citizen

D- only pharmacist can open

Expiry of compounded syrup contain water is?

A-7 days✔

B-9 days

C-14 days

-Before dispensing of TPN which drug should be discontinued?

A- Propofol
B- dexmetomidine

C- glycylcycline (tetracycline)✔

C- Fentanyl

Which the carrier in krebs cycle?

Oxaloacetat✔

Citrate

-Which one of the following question's types is used to identify a certain piece of information?

A)Leading questions

B)Open ended questions

C)Closed ended questions✔

D)Process and outcome questions

Which of the following medications can cause pneumonia as a side effect? A) Lisinopril B) Metformin
C) Simvastatin D) Omeprazole✔

Case patient has hypertension and diabetes and lab results of randomly glucose and HbA1c all were high
, which anti hypertension should take?

-amlodipine

-Candesartan✔

-Bisoprolol+ irbesartan

-Hydrochlorothiazide+ lisinopril

!antidot of opiods

.1naloxon orall

2.naloxon s.c

3 .naloxon 400mg iv✔

4 .Naloxon 400(mcg)im
The pKa of acetyl salicylic acid (aspirin) is 3.5. The pH of gastric juice in human stomach is about 2−3 and
the pH in the small intestine is about 8. Aspirin will be:

A-unionised in the small intestine and in the stomach

B-completely ionised in the small intestine and in the stomach

C-ionised in the stomach and almost unionised in the small intestine

D-ionised in the small intestine and almost unionised in the stomach✔

1 )The Drug Enforcement Administration (DEA) regulations require pharmacies to keep


(controlled)substances records, including prescriptions for at least?

a) two years

b) three years✔

c) four years

d) five years

[Controlled 3year / Narcotics 5year]

Drug of choice adrenal insufficiency?

Hydrocortisone iv✔

Cortisone oral

Preventive and curative institution is?

A. primary✔

B. secondary C. tertiary

D. specialty

Which of the following could be stored in a freezing unit?

A. Insulin

B. Interferon

C. varicella vaccine✔
D. Human growth hormone

Should drugs be freezing? A. Interferon

B. Vaccine ✔ C. solution

Phenytoin metabolism reaction is?

A- Oxidation

B- decarboxylation

C- 4-hydroxyphenyl✔

D- methylation

case about a diabetic patient type 2 he takes metformin but his blood sugar is so high and uncontrolled
and hb1ac was very high .. what to take?

A- insulin glargine + insulin asprt✔

B- insulin glargine + glyburide

case E coli with CKD?

A. Ciprofloxacin 400 iv every 12 hrs

B. Ciprofloxacin 400 iv every 24 hrs✔

C. meropenem 500 iv every 8 hrs

D. meropenem 500 iv every 12 hrs

pt DM2 symptoms ..when to lose weight..HTN ,heart failure ..do life style change and excises drug
choice for DM 2?

A- Insulin

B- liraglutide‫✔االصح‬

C- metformin (first line)

D- Glyburide
Which of the following is the result of drug interactions between birth control pill and erythromycin?

A. The effectiveness of erythromycin may increase

B. The effectiveness of erythromycin may decrease

C. The effectiveness of the birth control pill may increase

D. The effectiveness of the birth control pill may decrease✔

patient with major depression should receive antidepressant therapy for at least?

A- 2 weeks

B- 6 weeks

C- 2 months

D- 6 months✔

case about patient with runny nose and complain about and need fast relief of congestion?

‫ ينفع مع‬A. pheniramine✔ runny nose

congestion ‫بس ما ينفع مع ال‬

‫ ينفع مع‬B. pseudoephedrine $ congestion

runny nose ‫بس ما ينفع مع ال‬

‫ ما ينفع مع االثنين اال اذا كان السبب حساسيه‬C. intranasal cromolyn

‫ ينفع مع االثنين اذا كان السبب حساسيه‬D. corticosteroid

What is the secondary prevention?

A. Vaccination

B. Immunization

C. Do exercise

D. Blood test✔
Influenza virus?

A-Sporadic

B-Pandemic

C-Epidemic✔

-[Covid-19] : pandemic✔

What's (Z) in statistics?

A- dependent

B- independent

C- sample size

D-Standard score or standard deviations✔

To which of the following the major histocompatibility complex unique?

A) Each cell

B) Each organ

C) Each species

D) Each individual✔

Patient has many comorbidities, liver failure,

renal impairment, gout, and has partial seizure, what do u recommend antiepileptic that works for his
condition??

A. Valproic.

B. Phenytoin.

C. Gabapentin✔ .

D. Ethosuximide.

dizziness when he stands, the responsible drug is?

A/Hydralazine

B/isosorbide dinitrate
C/ prazosin✔

‫اذا مو موجود نختار‬A

What is the early sign of breast cancer

-lump✔

-nipple discharge

-Nipple retracted

-Dimpling

Dr prescribed for pt with open-angle glaucoma eye drops one drop/day and told him to administer?

A. At night✔

B. At morning

Patient with heart failure NYHA class IV and difficulty breathing, comprehensive history was given and
medication list was

given with doses of lisino and metoprolol and furosemide 10 mg (all were low doses)

A. Patient has suboptimal doses of heart failure medication

™ Which of the following is evidence-based treatment of aspirin?

B. secondary prevention of patient with MI✔

C. primary prevention of stroke to a 33-year-old make

D. other choices were primary prevention with specific patient age and disease mentioned

which the fallowing uses in prophylaxis stroke

A. anticoagulant

B. thrombolytic drug

C. antiplatelet✔

‫ ؟‬how to prevent stroke ‫ وسوالف كثير بعدين في االخير يقول‬antiarrhythmic ‫جابوا كيس غريبة شوي تقول مريض‬

: ‫الخيارات‬
Anti platelet

Anti arrhythmia

Anti coagulant✔

A lactating mother comes to emergency department complaining of nausea and vomiting. Which of the
following agents is most appropriate to give?

A. Cyclizin

B. Domperidone✔(increase prolactin level)

C. Dimenhydrinate

D. Levomepromazine

‫لو‬pregnant : A + B6

Patient is with difficulty in urine, he was previously with urine incontinence, he is feeling dizziness and
hypertensive, his bp is 140/90. He is taking Lisinopril, HCTZ and Oxybutynin, which is possible
intervention?

A. DC Oxybutynin✔

B. DC lisinopril and Oxy

C. DC HCTZ and Oxy

D. Continue all

324 -A non-smoker patient suffers from wheezing and chest tightness, what medication

he should use?

A. B2 agonist✔

B. corticosteroids

C. omalizumab

Memantine mechanism of action in Alzheimer?

A. affinity uncompetitive N-methyl-D-aspartate (NMDA) receptor✔

B. Memantine also blocks the 5-hydroxytryptamine-3 receptor (at

a potency similar to the NMDA receptor


C. And thus preventing loss of neurons

A 6-year-old boy is brought with complaint of uncontrollable spasm-like movements and excessive eye
blinking for four months. Chronic motor disorder was diagnosed. Which of the following is the best
management?

A. pregabalin

B. Haloperidol✔

C. Methadone

D. Chlorzoxazone

Which drug should be used for both bone and GIT problems?

A- Ca gluconate

B- Ca carbonate✔

C- Metoclopramide

D- Omeprazole

tetracycline‫لما حصل له ميتابوليزم اتوقع ويقول هل في‬

Acidic medium✔

Neutral medium

Basic medium

‫ ويسأل كيف تأثيرهم على بعض‬enteric coated aspirin ‫ مع‬amitriptyline ‫جايب دوا‬

A- Slow absorption of aspirin

B- Slow absorption of amitriptyline✔

C- Increase absorption of amitriptyline

Sterile preparation is a very complex manufacturing procedure that

requires -23 attention to the ingredients added in it. Which of the following

ingredients can be added when preparing a sterile drug?


A. Antioxidants

B. 2% Benzyl alcohol

‫ ✔✔االصح‬C. Bactericidal agents

D. 1.9% Sodium Chloride

Which is responsible for coloring the plant?

A. Tannin

B. flavonoid✔

C. alkaloids

D. coumarins

Role of tetrahydrofolate?

Acyl carrier

Nitrogen carrier

Electron carrier

Carbon carrier✔

Genomes of bacteria?

A-DNA✔

B-Rna

C-DNA OR RNA

D-DNA AND RN

-Genome of viral

A-Dna

B-Rna

C-DNA OR RNA✔

D- DNA AND RN

-HIV?
A-Dna

B-RNA / Retro virus✔

C-DNA OR RNA

D- DNA AND RNA

To decrease side effect of metformin on GI?

Divide dose

Take it with PPI✔

A 28-year-old woman presents with tachycardia and palpitations (ventricular ectopic beats). History
revealed essential hypertension and asthma. Patient wants to conceive (mean become pregnant)

BLOOD PRESSURE: 147/98mmHg

What is the most likely alternative to the β-blocker in terms of efficacy on pressure, heart rate and
relative safety?

A. Prazosin

B. Enalapril

C. Diltiazem

D. Methyldopa✔

Drugs cause Oligohydrosis?

Phenytoin

Topiramate✔

Zonisamide

Oxcarbazepine

The action of Digoxin reduced by:

Antacid‫✔ األصح‬

Hypokalemia

Hyperthyroidism
Verapamil

Hypothyroidism

Maximum dose of ergotamine for day?

10

6✔

Captopril contraindication?

In bilateral stenosis✔

History of angioedema✔

Pregnant✔

‫كلها صحيحه ما تجي بنفس الخيار‬

-carrier in TH ( T helper cell contain)?

A- cd4✔ -

B- cd8-

-T cytotoxic : cd8✔

Glargine insulin take?

1 -morning

2 -evening✔

3 -after meal

4 -before meal

] Long is take night[*

] Rapid is take before or after food[

]Short is take before meal 30 min[


What type of drug leading nightmare in sleep?

-Beta blocker-antihistamine 1gen✔

-diphenhydramine✔

⁃Azathioprine significantly interact with? allopurinol✔

erythromycin

Saudi MOH allow advertisement of drug in

-magazine

-Social media

-Medical conference

-Private hospitals and clinics✔

-PUFA need important vitamin?

Vit E✔

-Vitamin A in?

A-Fish oil

B-cod liver oil✔

Side effect of Refeeding syndrome?

Hypophosphatemia✔

Hypocalcemia✔

Which of the following statements about digitalis associated emesis is true?

A. It is due to Na/K-ATPase inhibition within the stomach✔ .

b. Itis due to stimulation of chemoreceptor trigger zones.

c. It is only seen with oral administration.


d. It is only seen with rapid IV administration

Eye preparation sterile?

hypotonic

hypertonic

isotonic✔

Essential fatty acid most source from?

plant ✔ or milk or water

Validity of pharmacy permit is ‫?صالحية ترخيص‬

5-years5 ‫ اصبحت االن‬3 ‫✔ كانت‬

Anticancer activity of Doxorubicin?

Intercalating‫✔ طلب عمله‬

Alkylating

Antimetabolite

Antibiotics‫< لو طلب التصنيف‬

non aqueous preparation?

A. Syrup, colloidal, mucilage

B. Essence, colloid and elixir✔

-Advantage of pseudoephedrine over ephedrine?

‫ ✔ اقل فعاليه بكثير‬Pseudoephedrine having considerably less effective

-Ephedrine to psuephdrin.??

To use it in nasal congestive✔


9 )Which of the following is an indication for methotrexate:

A/worts

B/times infection

C/psoriasis✔

D/acne

8 )Which of the following as an indication for clomiphene citrate?

A.Dysmenorrhoea

B.depression

C.infertility✔

D.Nausea

4 )Which of the following statement about digitalis is true?

a.Increase heart rate

b.Decrease myocardial contractility

c.Decrease preferences resistance

d.Increased cardiac output✔

5 )Which of the following is correct explanation of tachycardia that occurs after nitroglycerin ingestion?

A/Reflex sympathetic discharge✔

B/Decreased intracranial pressure

C/Direct positive chronotropic

D/Increase norepinephrine secretion from intracranial nerve ending

6 )Which of the following medications is not effective for the treatment of P.aeruginosa UTI?

a.TMP/SMX

b.Ciprofloxacin

c.Methenamine✔

d.Norfloxacine
7 )What is the rationale for the preferred use of inhaled corticosteroid over oral corticosteroids and
treatment of asthma?

A. Increase efficacy

B. decrease systemic side-effect✔

C. increased the ease of use

Complication of liver diseases and cirrhosis?

A-Portal hypertension

B-Hepatic encephalopathy

C-Ascites✔

Gelatin test came positive with?

A-Protein✔

B- Meconioc acid

C-Carbohydrate

You received a prescription on a plain paper what to do?

A. Tell the manager

B. Talk to the prescriber and don't dispense✔

Potent glucocortisone drug?

Hydrocortisone

Methylprednisolone

Betamethasone✔

The cause of lower adherence in diabetic patient?

Medication cost✔

Acid formulation
Alkaline formulation

Which of the following ranges of volume of distribution is deduced for warfarin?

(anticoagulant) that is 98% bound to proteins?

A. 3-5 L✔

B. 40-50 l

C. 50-500 l

D. 5000-10000 l

Crcl‫ايش اقيس فيها‬

1 -globular filtration✔

2 -renal secretion

Used in renal function

1 -crcl✔

2 -BUN

-drug has a volume of distribution of 35L in a 70 Kg man.Which of the following best describes Its
distribution?

A. it is bound to DNA

B. it is dissolved in lipids✔

C. it has low bioavailability

D. it is mostly distributed in plasm

-Thoracotomy indications?

Iodine solution✔

Or Propylthiouracil

‫الباقي ماذكره‬
-Red man syndrome

Vancomycin✔

Streptomycin

-diphtheriae is caused by what bacteria? Cornebacterium diphtyeria

-Drug that will cause toxicity for the Baby in Breastfeeding mother ? depend on MCQ

myalgia induced by?

Statins

-Name of drug that if it was combined with statin will result in sever rhabdomyolysis?

•Gemfibrozil

-Vitamin given with iron to increase non-heme iron absorption?

•vit C

-Crohn's disease management:

•infliximab

-Class I bio pharmaceutical classification

•high solubility high permeability

-phenobarbitals MOA? acts on GABAA receptors increasing synaptic inhibition

-Methotrexate binding?

Irreversibly

competitively✔
non-competitive

-Phenytoin?

Linear

non linear✔

-CHADS2-VAS score is 5 what is the appropriate management? Oral anticoagulant

-anti arrhythmia drug that cause thyrotoxicosis?

•Amiodarone ✅ or lidocaine..

•Propranolol MOA ? non-selective beta receptor antagonist

-second mediator?

•serotonin☑? or cytokines ..etc

-What type of vaccine is the cervical cancer vaccine? Human Papilloma virus vaccine

-case about newly diagnosed HTN patient asking about the normal range of systolic and diastolic Bp?

80/115•or 170/90 or 140/70

-CI compilations in HTV drugs

•Lisinopril + valsaratan

-alkaloid will cause arrhythmia if it was taken iv large dose or something like that..

theophylline

-Fungi cell wall content?


Chitin

?Where you can find the antigens?

A- On the plasma cells✔

B- T-cells

C- B-cells > immunoglobulin / antibody

Which cells has Antibody on its surface?

B cell✔

T cell

T helper

T killer

Glycoprotein is found in?

A-on the surface of the lipid bilayer of cell membranes✔

B-Mamalian cell✔✔

‫االصح لو موجوده نختارها‬


C-E.coli

D-fungal cell

-showed structures and asked for one structure contained chiral centre..

-pseudomonas DOC ? Ceftazidime✅ depends on MCQ

-Definition of Category B in pregnancy ? Animal studies have revealed no evidence of harm to the
fetus

-MRSA treatment? Vancomycin


-management of status epilepticus? lorazepam or Phenytoin or valporate all are correct?☑

-DOC prophylactic malaria in pregnancy?

Chloroquine and mefloquine all are

-evaluation of COPD

•exacerbation

-treatment of Angle-closure glaucoma?

•timolol

-Ex on two compartments kinetic?

Liver✔

Plasma

Etc

-Amino acids are absorbed in ? the proximal convoluted tubule (PCT)

Volume of distribution (Vd) means?

•Conc of drug in blood Or

Equivalent distribution between tissue and blood

Conc of drug in tissue

Potent glucocortisone drug?

Hydrocortisone

Methyprednisolone

Betamethasone
Symmetric distribution ( no skew)

A) median , mood, mean the same✔

B) median and mean the same different

mood

C) median, mood, mean all different

D) mean and mode the same different

median

Traveller diarrhea treatment:

a)-Ciprofloxacin 500 mg bid for 3 days✔

B)-azithromycin 500mg single dose

Patient has accident and take anti psychotic with Cimetidine for heart burn and have panic attack s.e
happened?

A. cimetidine increase metabolism of drug

B. decrease metabolism✔

Decrease the duration and severity of disease:

A-primary prevention

B-secondary prevention✔

C-tertiary preventi

Least interaction with grapefruit.

simvastatin

pravastatin✔

lovastatin

Rosuvastatin

Pilocarpine used in all of the following except:


a)Acute attack of closed angle of

glaucoma

b) Standard drug for close angle glaucoma

c) Used alternatively with mydriatic to separate the iris✔

d)Standard one for open angle glaucoma

-Side effects of short half life hypnotics?

Rebound insomnia✔

Develpment of tolerance

The master budget is a comprehensive financial planning

document. called?

-sales budget

-operation budget✔

-Organization plan

-Maximum dispensing for prescription of amphetamine from outpatient

department

30180 90 60✔

bradycardia and QRS prolongation?

ACEI, diphenhydramine, propranolol✔, amlodipine

A hospital conducted a pharmacovigilance (PV) activity to detected events after certain treatment, by
asking patients directly and patient records. Which of the following is the type of PV activity conducted
by?

A) Active PV✔

B) Passive PV
C) Mandatory PV

D)Spontaneous PV

A 23-year-old drug addict man was brought to the Emergency Room with severe agitation and
excitement. He was administered intravenous diazepam, with no improvement in his condition. Which
of the following is the mostly likely drug the patient was addicted on?

A.Opioids B.Cocaine

C.Pregabalin. D.Cannabinoids✔

The difference of prescription and herbal?

quality

need safet phase

need RCT✔

Newly graduated pharmacist hired in multinational.what would be his access of care?

A.General

B.Limited✔

Clinical pharmacist functions focus on

A. product

B. Hospital

C. Diagnosis

D. Patient✔

Who is currently responsible for financing and budgeting for healthcare services in KSA?

A. MOH✔

B. NUPCO

Manager wants to remove medications and wants to formulate a committee to assess and evaluate he
should assign who?
A. Planning B.Organizing✔

C.Controlling D. Leading

Alternative pathway if there is no glucose?

A. Ketones✔

B. Cholesterol

C. Triglycerides

D. Protein

Dispense the medications for chronic diseases every?

A. 3 months✔

B. one month

C.two month

A patient has diarrhea and the doctor wrote him a natural remedy to keep the fluids in order and so that
he does not lose more fluids, what did he advise him to do?

A/increase water intake

B/Sports

C/honey

D/Yoghurt✔

The presence of blood in vomiting indicates?

A/hemoptysis

B/hemateamesis✔

C/hematuria

D/hemacheziz

which patient population are at high risk of medication errors?

A- pediatric✔
B- adolescent

C- adult

D- childbearing age women

Patient with inflammatory acne asks pharmacist about cleansing product, what do you recommend?

A. hydrocort 1%

B. mild facial soap✔

C. hydroquinine

54-female patient on chemo started on cyclophos and anthracyclines what should do for her?

A-echocardiogram (ECG) ovaries preservation✔

B-acidification of urine alkalinization if urine

What is the main absorptive function of the colon?

A. Iron

B) Triglycerides

C) Water and glucose

D) Sodium, chloride, and water✔

elderly renal pt?

A-Ibuprofen

B- sulindac

C- indomethacin

D-codaine+paracetamol✔

Identify the best definition of planning.

A. An integrated process in which plans are formulated, carried out and controlled✔

B. Devising ways of achieving the objectives of an organization.

C. Setting an organization’s objectives and the means of reaching them.


D. The core activity of planners and planning departments.

-Immune system works an important role in cancer treatment by?

A. Macrophage

B. cytotoxic✔

C. complex

Which of the following is an intervention study?

A)Cohort

B) Ecological

C)Case-Control

D)Randomized Clinical Trial✔

In pharmacoeconomic studies, the costs (inputs) for types of analyses are measured in dollars but the
pharmacoeconomic method selected will depend on how the outcomes are measured.

Which of the following analysis described the method when patient (preferences for alternative)
treatments are being considered as the outcome?

A. Cost-Minimization B. Cost-Effectiveness

C. Cost-Benefit. D. Cost-Utility✔

Which of the following terms measures the association between the exposure and the

outcome?

A. P value

B.Risk ratio✔

C. Standard deviation

D. Confidence interval
One of the following medications is not affecting the potassium level?

A- salbutamol

B- digoxin✔

C- insulin

D- sodium polystyrene sulfonate

E- fosinopril

Which electrolyte disturbance lead to digoxin toxicity?

A. Na

B. K✔

C. Mg

D. Ca

C ‫لو غير موجود نختار‬

Which of the following cause digoxin toxicity?

A. Hypomagnesemia✔

B. Hypercalcemia

C. Hypophosphatemia

inhibits dihydrofolate reductase, preventing the reduction of dihydrobiopterin (BH2) to

tetrahydrobiopterin?

A-Adenine

B-Thymine

C-Guanine✔

D-Cytosine

Which of the following is considered a direct method to measure adherence?

A. Pill count

B. Pharmacy record
C. Patient questions

D. Body drug concentration✔

Which of the following should not be done for sound alike medications?

A. Use tall man letter

B. Verbal order✔

C. Non-verbal order

Case of patient with severe allergic rhinitis and needs steroid that given orally?

A. Hydrocortisone

B. Prednisolone✔

C. Betamethasone dipropionate

D. Beclomethasone

drug cause delay of empty stomach?

Atropine✔

Which of the following require monitoring?

A. IV UFH✔

B. SQ UFH

C. Enoxaparin

D. Fondaparinux

Hypertensive patients counsel?

A. Increase k intake✔

B. Increase zinc intake

C. decrease k intake

D. decrease zinc intake


advantages of CPOE system:

1 )reduction in medication errors✔

2 )process improvement

3)cost-conscious decision making, clinical decision support, and

efficiency

Which of the following is the best treatment for St. Aureus positive mastitis?

Cephalexin✔

Nitrofurantoin

Clindamycin

case asthma classification of patient has twice cough/w and no interference with daily activity?

A. intermediate✔

B. mild persistent

C. moderate president

D. severe persistent

What is the maximum beyond use date of repackage drug?

A. 3 months

B. 6 months✔

C. 9 months

-Narcotics prescription?

A-7 days✔

B- 3 days

C-1 month

D - 6 months

-Non narcotic ? 1 month✔


Patient came to the pharmacy with prescription that was prescribed 95 days ago, what should you do?

A. dispense the necessary medication only

B. dispense for one month only

C. ask the patient for the latest prescription✔

D. refuse dispense

1-A pregnant woman is travelling to a high risk malarial country which antimalarial

prophylactic agent caries the most fetal risk in pregnant women?

A. Mefloquine

B. Doxycycline✔

C. Chloroquine

D. Atovaquone-proguanil

Which of the following medications may alter the renal function?

A. isoniazid

B. gentamicin✔

C. acetaminophen

D. cyclophosphamide

64 -Which of the following local organization's primary mission is to the safety and quality of healthcare
in health institution?

A) Saudi central board for accreditation of healthcare institutions (CBAHI✔)

B) Saudi Commission for Health Specialties (SCFHS)

C) Saudi Food and Drug Authority (SFDA)

D) Saudi pharmaceutical Society (SPS)

Polystyrene role?

A-⬆ca

B- ⬆ k
C- ⬆ ca

D- ⬇ k (hypokalemia)✔

Rheumatoid arthritis take methotrexate and still not improved after 3 months?

A. add Adalimumab and abatacept

B. add HCQ (Hydroxychloroquine)✔

C. continue methotrexate

D. switch to another medication

Methotrexate contraindication in?

A - Pediatric

B - Osteoporosis

C- Breastfeeding✔

D- Heart Failure

42 )Which of following prescriptions may have a potential to cause medication error?

A. Amlodipine 5.0 mg PO OD✔

B. Thyroxin 25 mcg PO daily

C. Acetaminophen 500mg PO TID

D. Regular insulin 5 units subcutaneously STAT

To ensure safe and proper disposal of hazardous waste, a color- coded system was created. Which of the
following colors is the color of hazardous waste containers?

A. red

B. blue

C. yellow✔

D. orange

]Natcotic Red[*

]Controlled yellow[*
]Hazardous yellow[*

Which of the Following is required to consider an adverse drugs event report

A. drugs dose

B. patients tall

C. event name✔

D. reports phone

-Which of the following drugs require extra precautions during handing to ensure medication safety?

A. Methotrexate✔

B. Potassium

C. Dopamine

D. Thiamine

An ambulatory care pharmacist received a prescription for (liraglutide) (REMS), from Bariatric Endocrine
consultant as per medication under (REMS) should be provided to the patient with documents. Which of
the following is the meaning of (REMS) beside the name?

A) This medication has teratogenicity effect

B) This medication is under (restriction and evaluation medication Strategy)

C) This medication is under (Risk Evaluation and Mitigation✔ )

D)This medication is under (Restriction Evaluation Medication)

39-)A patient medication was changed from carbamazepine to phenytoin The technician pulled the 100
mg tablets of phenobarbital off instead of the 30 mg tablets. The pharmacist did not notice that when
the prescription was checked. The patient received a higher phenobarbital and was hospitalized for few
days for observation. Which of the following is the best statement concerning this situation?

A) This is considered as adverse drug reaction

B) The physician should have informed the pharmacist about prescription's change

C) The patient should be aware of what he was prescribed and the difference

D) This is preventable medication error if the pharmacy in safety measures✔


Which of the following medication error is difficult to prevent?

A-Omission error

B-Wrong time error

C-Incorrect duration of treatment

D-Allergic reactions to an unknown patient✔

hyperkalemia 6.8 mmol/L treatment,

1-Insulin and dextrose✔

2-Dialysis

which type of data the researchers collect from patients before submitting

the research (proposal)?

A- Survey✔

B- Primary

C- Secondary

D- Laboratory

Animal reproduction studies have shown an adverse effect on the fetus and there are no adequate and
well-controlled studies in humans,28- p value?

A- > significant clinical

B- < significant statical✔

C- > significant statical

D- < significant clinical

Anti-arrhythmic drugs used in supra-ventricular and Ventricular Arrhythmia

Sotalol

phenytoin

Flecainide✔
Procaine

Which IV container need filtered needle?

Vial

Bag

Ampoule✔

All IV preparation need filtered needle

-clinical pharmacist during the round they ask her to choose one antibiotic for C.diff,and after searching
she found that vancomycin orally bid is the best to choose ,from where she conduct

the information?

-expert opinion✔

-cohort study

- rct

24-year-old female is diagnosed with genital herpes simplex virus

infection. Which of the following agents is indicated for use in this diagnosis?

A. Valacyclovir✔.

B. Cidofovir.

C. Ganciclovir.

D. Lamivudine.

Patient who have history of MI and HTN and he is diagnosed with osteoarthritis and currently taken
acetaminophen 650 mg every 6 hours but still suffering from continuous knee pain . What is the
appropriate management?

1 -celecoxib

2 -Naproxen✔

3 -oral GLUCOSAMINE

4 -oral hydrocodone
compare the safety of one medicine with another medicine it would be necessary

to:

a) Review the evidence from Phase I clinical trials.

b) Review the evidence from Phase II clinical trials.

c) Review the evidence from a post marking clinical trial✔.

High sterilized substance can be kept in room temperature for how long?

A. Up to 24 hrs✔

B. 3 days

C. 45 days

Drug A stablity in ph ( 1 - 8) in which ph is less stable?

1✔

According to SFDA, what is the permitted age to dispense cold medications to children?

A. Up to 2 years✔

B. Up to 6 years

C.Up to 4 years

Drug need incremental dose for elderly?

Clomipramine

Losartan

Thyroxin✔

Lisinopril

iron interaction with?


mg

levothyroxin✔

metformin

Eye misaliment in children

-astigmus

-starpismus✔

-olyopia

Aspirin/ ibuprofen separation?

A-Take ibuprofen 4 hr before apirin dose

B-Take ibuprofen 8 hr before apirin dose✔

Plan b contraceptive take about?

A. 24 h B. 48 h C. 72 h✔

Buy inventory then reselling to customer?

Cost of good sell✔

Which plant is 5Ht3 activity?

Ginger✔

Age groups isinactivated polio recommended for?

Children up to 6 years old✔

Design could be deemed an study?

Clinical trial✔
What the main method transmission of brucella species?

Ingestion of unpasteurized milk/milk product✔

Immunoglobulin activates complement system?

A. IgG✔

B. IgM

‫ االطول مده‬immunoglobulin ‫؟‬

A ) IgG✔

b) IgE

c)IgM

Drugs exhibits dose-dependent pharmacokinetics at normal therapeutic doses?

Phenytoin✔

Vaccine give for all family?

Herd✔

Spical

Local

All the following diazoxide side effect except?

Diuresis✔

Tachycardia

Hyperglycemia

Hyperuricemia

Used to determine the significance of the result?

P value✔
BB with partial agonist?

Labetalol

Propranolol

Clonidine

Acebutolol✔

Patient culture is E.coli ( ESBL ) what you will give?

A. Ceftriaxone 500 mg IV Q12H

B. Ceftriaxone 500 mg IV Q 24 H

C. Meropenem 500 mg IV Q 8 H✔

D. Meropenem 500 mg IV Q 12 H

Acetyl coA and citric acid cycle which vitamin is used?

A. B6

B. B12

C. B2 ✔

D. B9

Bond between Carboxyl and amine?

A. hydrogen bond

B. b- ionic bond✔

C. vander wall

‫ نختارها‬peptide bond ‫لو موجودة‬

A 43-year-old man has abnormal liver function tests noted at a routine work related medical. He has a
history of angina.

Which of the following drugs is most likely to be responsible?

A. Atorvastatin✔

B. Amlodipine
C. Aspirin

D. Atenolol

What is considered moderate intensity statin?

Atorvastatin 20 mg✔

Pravastatin 20 m

? drug drug interaction ‫مين اقل‬

A- Simva

B- Atrova✔

Graph presented in the question. Calculate Therapeutic index in graph ED 100 , LD 400:

A) narrow therapeutic index✔

B) Wide therapeutic index

?equivalent ‫(إنهم الداتا من مجموعتين نقول متىعن االستاتستك‬50

a) if Mean and median and mode are equal✔

b) if Mean and median and mode are not equal

c)if Mean and median are equal but mode not

cells has role in inflammation?

a)Basophiles✔

b)Neutrophils

MOA of lactulose?

A- acidify colon and limit ammonia✔

B- alkalization colon and limit ammonia

which microorganism cause Lyme disease?


•virus

•Fungi

•Bacteria✔

•Parasite

women take Combination contraceptive and heavy smoker, no family history HF

or VTE

A) add statin

B) continue CC and add statin

C) use barrier contraceptive

D) use only progesterone contraceptive✔

-Which of the following strategies are used with low literacy patients to Encourage them to change their
behavior?

A. Keep information focused and organized

B. Use a normal speaking tone and volume

C. Use briefer educational sessions

D. Use more visual aids✔

-Which of the following strategies are used with older adult patients to encourage them to

change their behavior?

A. Use teach-back more often throughout the session✔

B. Introduce new concepts more slowly

C. Use briefer educational sessions

D. Use more visual aids


Second part:

Rate control for patient with ischemic heart diseas

( metapolol, bisoprolo ) ✅

which of the following preparations should be refrigerated regardless of it is ingredient?

A.eye drops

B.suppository ✅

C.oral solution

D.capsules

whats wrong with iron in geriatric patients

*decrease absorption * ✅

‫ ايش نوع الخطأ ؟‬، ‫الفني صرف دواء بدل دواء ثاني واكتشف الصيدلي ا بيصرفه ان فيه خطا قبل اليعطيه المريض‬

Dispensing✅

Drug need monitoring ?

✅theophylline

?Flash tech for which tablet ?

immediate release ✔

sustained release tablets✔

‫ نختارها‬fast ‫ولو كانت موجوده‬

- Virus associated diarrhea

lactobacillus ✅

? Geriatric calcium dose


1200 ✔

Ciprofloxacin + metronidazole for treatment ?

mild to moderate crohn ✅

Hormone cause uterine contraction during child birth ?

oxytocin✅

Drug inhibit xanthine oxidase ?

Allopurinol✅

Clozapin dose for female’s ?

5 ml✅

Antimetabolite drug ?

anti cancer ✅

Drug cause miosis and respiratory depression ?

opioid toxicity✅

First sign of breast cancer ?

painless lump✅

Tamoxifen use ?

treat breast cancer✅

HLAB 5702 ?

abacavir✅
Cholinergic antagonist?

Ipratropium bromide✅

Antiarrhythmic drug cause iodide hypothyroidism ?

Amiodarone✅

Boric acid function in eye drops ?

cleanse or irrigate the eyes ✅

Long acting insulin ?

insulin glargine 24 h✅

insulin degludec 42 h✅

.how long the control drug should be administered by IV rout ?

24 h✅

20 weeks pregnant woman suffering from dizziness and tiredness , lap results show low hemoglobin ,
what to do ?

Ferrous sucrose iv✅

Why give vasopressin with NE in septic shock ?

To prevent kidney necrosis ✅

Patient have heart failler + mucus in lung , difficulty breathing , what is the condition ?

Wet and cold✅

dissolve thrombus by formation of Fibrinolytic plasmin from plasminogen ?

Alteplase ✅
Streptokinase ✅

Lowest potency Glucocorticoids ?

Hydrocortisone✅

Phenytoin decrease IV dose monitor after how long ?

4 hr ✅

What not to use in IV ?

non sterile material✅

xl dosege?

slow release✔

sustained release

what will happen if you break down sustained release drug?

toxic✔

more absorbed

the past 7 years. Can she receive zoster vaccine

(ZOSTAVAX) now?

b. Yes, she needs to receive a dose of the

vaccine now, and an additional dose after

completion of chemotherapy to ensure a full

immunogenic response

c. No, she should not receive the vaccine due to her diagnosis of cancer and because she is

currently receiving chemotherapy✔

d. No, she does not need to receive the vaccine because it will not help since she has
experienced past outbrea

How long does it take before a flu vaccine

provides you with maximum protection against

the flu?

2 weeks ✔

Healthy children over what age should not

receive the Hib vaccine?

Five years✔

What part of an immunoglobulin is responsible for recognizing foreign things?

o Fc region✔

o Heavychain

o Lightchain

‫ نختاره‬Fab region ✔‫ لو فيه‬:‫*للتوضيح‬

Fc‫ ترتبط مع‬Antibody ‫*ال‬

Antigen‫ ترتبط مع‬light ‫*ال‬

Which one consider as physical examination?

- Regulatory

-Precaution✔

q- if the 1 mg of drug soluble what could be 20mg ?

freely soluble

very soluble✔

soluble

sparingly soluble
A 28 year-old woman has been raking fluoxetine for depression and now has worsening depressive
episodes.The resident doctor on the floor has been asked by the consultant ro stop fluoxetine and
initiate phenelzine.

How long should the washout period be before starting phenelzine?

A. 2 weeks

B. 3 weeks

C. 4 weeks

D. 5 weeks✔

Which medication requires negative pregnancy test before dispensing to woman of child bearing age ??

A. Labetalol

8. Prednisone

C. Ceftriaxone

D. Isotretinoin✔

What is the pharmacological category for insulin lispro?

A. Long acting insulin

B. Rapid acting insulin✔

C. Short acting insulin

D. Intermediate acting insulin

Which of the following medications require observing the patient for six hours with hourly heart rate
monitoring after the first dose?

A. Verapamil

B. Bisoprolol

C. Amlodipine

D. Fingolimod✔

Which of the following indications is appropriate for misoprostol?

A. Misoprostol is ineffective for preventionor treatment


B. Misoprostol is effective for both prevent ion and treatment

C.Misoprostol is effective for preventing NSAIDs induced ulcer✔

D.Misoprostol is effective for treatment of NSAIDs induced ulcer

A 15 month-old boy is brought to the Emergency Department by his parents with high grade fever,
vomiting, and non-blanching rash. He is started immediately on cefotaxime. The cerebral spinal fluid
culture reveals Neisseria meningitides.

What would be the recommendations regarding prophylaxis of his family and close contacts?

A.His parents should receive rilampicin

B.Antibiotic prophylaxis is not indicated

C.Patient's close contacts should receive rifampicin✔

D. Siblings under the age of 18 should receive rifampicin

In which of the following patient categories the (gentamicin) extended interval regimens (once daily
dosing) is most suitable?

A. Pregnancy

B. Burns patient

C. Gram Negative Infections✔

D. Enterococcal endocarditis

A 28 year-old woman with a history of seizures that is well controlled on valproic acid is asking for advice
regarding her medications.She is planning to start a family and is now taking folic acid. She wants to
optimize the medications before her pregnancy.

What would be the best recommendation?

A.Introduce phenytoin to valproic acid

B.Discontinue valproic acid immediately

C.Continue on valproic acid since

she is well controlled

D.Introduce levetiracetam slowly and titrate the dose of valproic acid to discontinue before pregnancy✔
Which of the following medications can be teratogenic if a pregnant woman handles crushes or breaks
the tablet?

A. Bosentan✔

8. Alfuzosin

C. Topiramate

D. Didanosine

dutasteride‫ نختاره وممكن يجيب بديل عنه‬Finasteride ‫لو فيه‬

Which of the following represents the immunity acquired by give living organism with

attenuated virus?

A) Local immunity

B)Passive immunity

C) Natural active immunity

D) Artificial active immunity✔

Attenuated mean?

Active virus ✔

Killed virus

Inactivated virus

A 60 year-old woman has been recently diagnosed with Stage II hypertension.

Which of the following medications would be suitable for her ?

A. Losartan/hydrochlorothiazide✔

B. Atenolol/nifedipine

C. Valsartan/furosemide

D. Amlodipine/triamterene

2D6 poor what to give?

Tamoxifen
Raloxefine✔

Which of the following antibiotics is used in surgical prophylaxis of colorectal surgeries?

Ampicillin

Cefoxitin✔

Linezolid

Augmentin

moderate physical activti?

swimming

basketball

brisk walking✔

Which of the following of the hypertensive therapies is preferred to be used

during the acute phase of ischemic stroke if elevated blood pressure should be

treated?

A)lisinopril

B)nicardipine✔

C)hydrochlorothiazide

D) metoprolol succinate

Erythromycin and birth control interaction

o Increasebirthcontroleffect

o Decrease birth control effect o IncreaseErythromycin

o DecreaseErythromycin

o No interaction ✔

- Mortality of children was 8 in 2012 and 6.4 in 2017, This mean?

A- More hospital open


B- Improve health

C- Nothing, more information need✔

72 kg Patient diagnosed with a fib and travels a lot do not have time what to give?

A. warfarin 5 mg

B. apixaban 2.5 mg bid✔

C. enoxaparin 40mg

D. enoxaparin 70 kg bid

Which of the following drugs is associated with progressive multifocal leukoencephalopathy side effect?

Bisoprolol

Pregabalin

Doxycycline

Natalizumab✔

Which one of the flowing best represents a direct medical cost?

A-pain

B- Transportation

C- Mortality

D- Medical professional time✔

Patient with severe gastric pain to right upper quadrant with vomiting also pain to back also get worse
with fatty meals ?

A/pancreatic✔ B/gallstones✔ C/hepatic disease

‫ صحيحه‬A ‫ بس انا عن نفسي اشوف‬B ‫االغلب قال‬

Which of the following is the Best and safest laxative for chronic constipation ?

A. Bisacodyl

B. Senna
C. Glycerin suppository

D. Metamucil (psyllium) ✔

C‫ نختار‬Acute ‫في حالة ال‬

Pt medications : Genvoya, insulin, amlodipine, spironolactone, Calcium gluconate. What is

theinteraction ?

a- amlodipine with Ca✔

b- Genvoya with Ca

c- Genvoya with amlodipine

Secondary mediator in type 1 hypersensitivity

A/T cell

B/ B cell

C/ cytotoxic

D/mast cell✔

which one cause mononucleosis?

A-Epastin✔

B-hymofylus

C-salmonella

Clinical pharmacist during the round they ask her to choose one antibiotic for C.difficle and after
searching she found that vancomycin orally bid is the best to choose, from where she conducts the
information??

A. Expert opinion.✔

B. Cohort study.

C. Retrospective.

Which of the following is a B-lactamse enzyme?


A) Plasmid

B) Pyrogens

C) DNA gyrase

D)Carbapenems✔

Drug should not use be take with ciprofloxacin?

acei

ferrous sulfate ✔

atorvastatin

metformin

Which of the following drugs has a high abuse potential?

A) nifedipine

B) allopurinol

C) varenichine

D) methylphenidate✔

28-year-old woman who is 8 months pregnant is taking naproxen 500 mg twice daily for the past four
weeks to treat severe backache. Which of the following health issues at the time of birth puts the
neonate at higher risk?

A. asthma

‫اقرب شي‬B. cardiac arrhythmias✔

♀‫برضو صحيحه‬C. uncontrolled bleeding

D. gastroesophageal Reflux Disease

which the following treatment Meningitis empiric treatment ?

A. Vancomycin

B. Ceftriaxone 2g 12 h IV 7-14 day✔

C. Ampicillin
D. Azithromycin

Calculation question: patient take warfarin and stop at 2:00 am warfarin T1/2

is 1.5 hr , what is the time needed to go to surgery?

A) 16:00✔

B)22:00

C)3:00

What is the best time to do lithium levels after the dose is administered?

A.4 hours

B. 6 hours

C. 8 hours

D. 12 hours✔ (It usually 8-12 h)

muscle numbness on his mouth?

hypophospteamia

hyperphsotamia

hypocalcemia✔

hypercalcemia

Which of the following is a late adverse

effect of lithium?

(A) nausea

(B) hand tremor

(C) seizures

(D) hypothyroidism✔

Which of the following factors may increase lithium concentration?

(A) caffeine
(B) osmotic diuretics ✔

(C) increased fluid intake

(D) nonsteroidal anti-inflammatory drugs

A pregnant woman has sulfa allergy, she suffers from vaginal itching her analysis shows positive E.coli

what is the antibiotic of choice for her infection?

A-Ciprofloxacin

B- Septazole

C-Nitrofurantoin ✔

D-Tetracycline

morphine refill?

0✔

3) part of the nerve and has role in covering and protecting the neuro impulse and conducting it ?

a) Axon

b) Mylene sheath✔

c) Soma

d) Dendrites

Contraindicated in breastfeeding?

A. warfarin

B. Lithium✔

C. insulin

D. none of the above


Which of the following drugs is a common cause of interstitial nephritis?

(a) Metformin

(b) Ranitidine

‫(اتوقع األصح‬c) Lithium ✔✔

(d) Omeprazole ✔

(e) Ondansetron

What is the Intestine PH range?

A. pH 5 to PH 6

B. pH 6 to pH 7.4✔

C. PH 7.4 to PH5.7

D. PH 5.7 to pH 6.7

Best diuretic in ADHF ? (Acute decompensated heart failure)

A. furosemide✔

B. spironolactone

C. HCT

Which of the following is Filamentous anaerobic bacteria?

A. Pseudomonas aeruginosa

B. Actinomycetes✔

C. Mycobacterium tuberculosis

Which of the following is the most rapid relief of acidity?

A. Omeprazole

B. Antacid✔

C. Sucralfate

D. H2 receptor antagonist
Which drug we can use it before niacin to prevent flushing?

A. Vitamin C

B. Aspirin✔

C. Ibuprofen

86- A 55-year-old man comes to the pharmacy complaining that has acetaminophen with codeine he
started 2 days ago to manage has post- operative pain does not seem any better than when he uses
acetaminophen alone. After the pharmacist reviews his profile, he noticed results from
pharmacogenomics testing performed 3 years ago that shows he is a CYP2D6 poor

metabolizer.Which is the best explanation why this patient does not seem to benefit from codeine?

A. Codeine excretion is rapid

B. Codeine absorption is slower ✔

C. Codeine is metabolized faster

D. Codeine dose should be increased

‫الخيارات كانها خطا‬


‫ الصح‬metabolized slow ‫ النه‬cyp2d6 low ‫ وذا اللي بيعمل‬metabolism

Food not used with amiodarone?

A. Watermelon

B. Grapefruit✔

C. Green tea

D. Pomegranate

Which cephalosporin has no antipseudomonal activity and should not be used in the management of
pseudomonas

aeruginosa pneumonia?

A. Cephalexin✔

B. ceftazidim

C. cefipime

D. none of the above


Which pharmacokinetic parameter estimates the renal function?

A. serum creatinine

B. intrinsic ciearance

C. blood urea nitrogen

D.creatinine clearance✔

TCA act on all of the following receptors expect:

a) Histamine receptors

b) Cholinergic receptors

c) Alpha receptors

d) Beta receptor✔

Prophylaxis from migraine aura?

A. Ergotamine 4 mg

B. Sumatriptan 50mg BID

C. Propranolol 20mg BID✔

Case patient take imipramine and asking TCA used in all of the following expect:

a) Panic attack

b) Schizophrenia✔

c) Migraine

d) Neuropathic pain

What drug should stop before TPN?

Fentanyl

Tetracycline ✔

Which of the following is characteristic of BBB?


A) passive diffusion of glucose-independent insulin ✔

B) allow cross glucose - depend on insulin

C) allow crossing polar drugs

D) prevent non polar drugs

Substance released from the liver in response to IL-6 secretion ?

A- CRP ✔

B- IL-2

C- Immunoglobulin

Which of the following is the major risk associated with the use of over-the- counter drugs?

A. Increased cost without benefit

B. Decreased drug-drug interactions

C. Increased potential for misuse and abuse

D. Increased risk of adverse events✔

Women newly get pregnant and preform diabetic test in the beginning of her pregnancy negative, what
is your recommendation

regarding gestational diabetic test?

A)No need for testing

B) Do another test between 4 and 12 weeks

C) Do another test between week 20 and 24✔

sever depression and anxiety can’t give patient ?

A- mirtazapine

B- buspirone ✔

C- Fluoxetine

- Phenobarbital prescription as outpatient for how many days ?


A. 30✔

B. 60

C. 70

D. 90

-Pt wit metabolic alkalosis and edema use ?

A- ringer

B- isotonic

C- acetazolamide✔

259- Which of the following prevent renal injury ?

A- Vit C

B- Vit D✔

C- i.v fluid

-what is the drugs we do not give old people ?

A- amlodipine

B- metoprolol

C- Metoclopramide✔

what is the recommended dose of IV N-acetyl cysteine for 6 years child ?

A. 2_5mg/kg

B. 20-15mg/kg✔

pharmacological group cause hepatic injury?

• Purine,

• pyrrolizidine ✔

dihydropyridine
Women during pregnancy took high doses of ampicillin

and after birth, the baby has high bilirubin. What is your intervention

‫ممنوع للرضع اقل من شهر‬ a) Ceftriaxone

b) Gentamycin

c) No antibiotic needed✔✔ ‫ذي أصح‬

Which ONE of the following is NOT a reason for intentional non-adherence

a. Reducing doses to avoid side effects

b. Forgetfulness ✔

c. Concern about addiction

d. Belief that the medication doesn’t work

e. Unable to afford the prescription charge

What is more effective method to prevent pregnancy ?

A. Oral contraceptives

B. Latex Condoms ✔

C. Spermicides

‫أصح‬D. intrauterine device ✔✔

‫ قالو ان‬d ‫أصح منى نصر واذا ماجابها وجاب التعقيم برضو األقوى‬

best way to collect volatile oil ?

A- gas chromatography ✔

B- liquid chromatography

Plague transmitted by

a) House flies

b) Fleas ✔

c) Mosquitoes
-Phenytoin Side effects except:

Gingival hyperplasia

Acne

Hirsutism

Nystagmus

‫أصح‬Nephrotoxicity✔✔

-Steven-Johnson syndrome

Tonic-colonic seizures DOC is:

a) Phenytoin

b) Ethosuximide

c) Valproate ✔

Pharyngitis, penicillin allergic, organism sensible to

bacitracin:
a) Azithromycin✔

b) Tetracycline

c) Cefaclor

d) Amoxicillin

Used in treatment of AIDS:

a-Acyclovir

b-Rifampicin

c-Zidovudine✔

d-Prednisolone

Nephrotoxicity is the side effect of:

a- Paracetamol
b-Metronidazole

c- Vancomycin

d- Gentamycin✔

Hepatotoxicity is the side effect of:

a- Paracetamol✔

b-Metronidazole

c- Vancomycin

d- Gentamycin

What is the major complication of morphine overdose?

a – AV-Block

b- Respiratory depression✔

c-Cardiac Arrest

d Liver dysfunction

Which one of the following is considered ultrashort acting barbiturate?

a- thiopental✔

b- secobarbital

c- amobarbital

d- phenobarbaital

Which one of the following is considered long acting barbiturate?

a- thiopental

b- secobarbital

c- amobarbital

d- Phenobarbital✔

Pseudomembranous colitis can be treated by


a- oral vancomycin plus metronidazole✔

b-vancomycin plus tetracycline

c- ampicillin plus metronidazole

d- amoxacyillin plus metronidazole

Which aminoglycoside antibiotic can be taken orally?

a-gentamycin

b-neomycin✔

c-amikacin

d-tobramycin

Patient forget her aspirin tablet in bathroom what will happen to the tablet ?

A/oxidation

B/ Reduction

C/ demethylation

D/ Hydrolysis ✔

A ‫اذا مو موجوده نختار‬

Pt has lung infection and use levofloxacine his INR is 5 this fluctuation considered as which

Interaction ?

A/drug disease

B/drug lab

C/ drug -drug✔

What's the variation in gene called?

Allel✔

Genotype

Phenotype
Which antiepileptic drug causes nephrolithiasis?

A. Zonisamide✔

B. Oxcarbazepine

C. Rufinamide

-Vaccine only oral?

Rota✔

-Vaccine taken orally or Im ?

Polio✔

New diseaes affect people and specific time?

A. Mortality

B. Morbidity

C. Incidence✔

D. Prevalnc

New born with fever,dyspnea ,and Jaundice. He can take?

A. Ampicillin and Gentamicin✔

B. Ceftazidime ang Gentamicin

C. Ampicillin and Ceftriaxone

Which of the following is the best treatment for st.aureus positive mastitis ?

Cephalexin ✔

Nitrofurantoin

Clindamycin

- Before 6-week Pt did surgery and he wanted to lift the hospital, which discharge

vaccine should he give?


A. meningitis

B. influenza

C. pneumococcal✔

-(Case) diabetic patient needs drug for

hypertriglyceridemia?

A. Statin

B. Glifibrozide✔

C. Cholestyramine

D. Eztimibe

which of the following drugs passes into mothers milk due to high lipid solubility?

A) diazepam✔

B) methyldopa

C) promethazine

D) dihydroergotamine

Treatment patient with acute crisi anemia

A. Morphin

B.Hydroxyurea✔

Metabolic alkalosis + edema treatment?

A. Ringer lactate

B. Isotonic solution

C. Acetazolamide✔

Reduce of atherosclerosis?

Green tea

Olive oil✔
QT segment time:

400 seconds

0.4 seconds ✔

0.004 seconds

4 seconds

IM injection:

3ml or 5ml✔

drug used after trial in hospital ?

A. internal validity

B. external validity ✔

C. reliability

Anti-diabetic drug used in pregnancy?

A- rosiglitazone

B- Metformin ✔

C- glargine is long acting insulin

glargine CI in pregnancy ‫للتوضيح‬

‫لو نوع انسولين ثاني نختاره‬

- blood pressure in neonate?

A. same adult

B. Higher than adult

C. Lowed that adult✔

Morphine to heroine??

A- Carboxylation
B- Esterification✔

Which medication can cause orthostatic hypotension? A) prazosin✔ B) donepezil

C) rivastigmine D) spironolactone

-Metabolic acidosis what is decrease?

A- Pco2

B- HCO3 ✔

C- CO2

-Metabolic acidosis what is increase?

A- Pco2

B- CO2 ✔

C- COH

Which one of the following questioning techniques uses hints and aid patients in answering questions?

A) Changing questions

B) Empathic responding

C) Prompting while asking ✔

D) Redirection the questions

Least drug interact with BB??

a) phenytoin

b) phenobarbital

c)rifampicin

d)fluoxetine✔

Which governmental agency gives approval to generic media enter the drug market in Saudi Arabia after
completing all requirements?

Ministry of Health (MOH)


Saudi food and drug Authority (SFDA)✔

Saudi Commission for Health Specialists(SCFHS)

use of caffeine in neonate where to look?

Drug in pregnancy and lactation

Hospital formulary drug

Harriet lane handbook ✔

-Cell that produces plasma cells?

T cell

B cell ✔

Macrophage

-activator receptor of mast cells ?

B2

Siglic

Platelet activator receptor✔

-Ceftriaxone:

Adipose tissue, brain, liver✔

-Ciprofloxacin:

Adipose tissue, liver, brain ✔

Piggyback mean ?

TPN

iv ✔

Which detoxification pathway is active paracetamol toxicity?


A. Oxidation

B. Reduction

C. Conjugation with glucuronide

D. CYP-450 dependent glutathione✔

Drugs prolonging QT interval:

Methadone ✔

Amilodepine

Linezolid

‫خيار رابع نسيته‬

Drug prolonging QT interval :

Amilodepine

Citalopram ✔

‫خيارين تاني نسيتهم‬

Which contain sugar part and non sugar part ?

Glycoside ✔

Glucose

Protein

-A patient is found to have been lacking in functional CYP2C19 enzyme activity. What would be your
recommendation?

‫األصح‬a. He should not receive Clopidogrel✔

b. He should not receive any CYPC19 enzyme inhibitors such as omeprazole or fluoxetine

c. If clopidogrel is administered, he will have increases INR

d. He is an ideal candidate to receive Clopidogre

Tablet what type of risk?


Mediums

High

Low✔

Bioavailability rate limiting step in tablet after reducing particle size-171:

-Absorption rate limiting step

-Dissolution rate limiting step

-Permeability rate limiting step

‫ في االختيارات حتكون الصح‬Disintegration✔ ‫لو فيه‬

B for solutions not tablet

Pt with AF and torsade de point which anti arrhythmic ues for this pt?

1- amiodarone

2-varpamil✔

3-defolitde

4-sotalol

A fishing scenario that is not preparing a report on expenses and profits. What type of report is it?

-Revenue

- Balance sheet

- Income statement

- Cash flow✔

Which correct?

A. Ergocalciferol is D3

B. Cholecalciferol is D2

C. Vit D 2 and D3 active biological

D. Vit D 2 and D3 inactive biological✔


2 years skin infection with penicillin

1-doxycycline

2-Amoxicillin +Clavulanic acid

3-sulphamethoxazol+trimethoprime✔

Lipinski 5 oral dosage form ?

A - Molecular mass less than 500✔

B- Hydrogen donar

C- Hydrogen acceptors

D- Partition co efficient

When evaluating patients with angina for possible development of nitrate tolerance, true statements
include

I. Tolerance is rarely associated with isosorbide dinitrate

II. Tolerance doesn’t occur in patients who are concurrently on metoprolol

III. Tolerance is dependent upon the administration schedule of nitrate medication

a. I only

b. III only ✔

c. I and II only

d. II and III only

e. I, II and III

Parenteral products with an osmotic pressure less than that of blood or 0.9% sodium chloride are
referred to as (

A) isotonic solutions.

(B) hypertonic solutions.

(C) hypotonic solutions. ✔

(D) iso-osmotic solutions


What is the best hypnotic for geriatric ?

A. Alprazolam ✔

B. Diazepam

C. Phenobarbitone

NSAIDs has the gastrointestinal side effect?

Naproxen

Meloxicam

Ibuprofen✔

Diclofenac

Which of the following capsules size has larger size suitable for human patients?

A- 0

B-1

C- 5

D- 000✔

* 5 : smallest / 000 : largest*

-Cockcroft gult use to measure creatinine clearance in?

A-acute kidney disease

B-stable kidney disease✔✨

Where in the cell is cytochrome P450 located?

a) Mitochondrial inner membrane

b) Cytoplasm

c) Mitochondrial matrix

d) Endoplasmic reticulum✔✨

Which of the following proteins transports foreign molecules out of cells?


a) Mixed function oxidase

b) UDP-glucuronosyl transferase

c) Glutathione-S-transferase

d) Multidrug resistance P-glycoprotein✔✨

What enzyme, or combination of enzymes, protects cells against superoxide generated in oxidation
reactions?

a) Superoxide dismutase

b) Catalase

c) Superoxide dismutase plus catalase✔✨

d) Glutathione peroxidase

-Fick’s law ‫?تتبع ايش‬

Diffusion✔

-Noyes Whitney?

Dissolution✔

-Henderson Hasselbatch?

PH✔

Reaction that convert pentazocine to

morphine?

‫ممكن نقل السؤال غلط‬


:‫الرسمه بالسهم من مورفين لـ بينتازوسين‬

A.Extention

B.Simplification✔

C.Rigidifacation

C‫ لو من البنتازوسين لـ مورفين نختار‬:‫*للتوضيح‬


Which of the following is the type of studies considers data from multiple studies of different designs to
draw conclusions?

o A. Case Series

o B. Double-blind study

o C. Systematic Review✔

o D. Consensus statement

Researchers compare a group of people who have diabetes with of people who have no diabetes, and
looks back in time to see characteristics of the two groups differ.

Which of the following is the described study design?

o A. Prospective cohort

o B. Case-Control✔

o C. Experimental

o D. Randomized clinical

Which one of the following designs could be deemed an study?

o A. Cohort

o B. Clinical trials✔

o C. Case-control

o D. Cross-sectional

A 61-year-old was recently diagnosed with parkinson’s disease He was

prescribed an anti-parkinson medication. Later, he was presented to the clinic

with mydriasis and narrow angle glaucoma. Which of the following medicationwas prescribed?

A. Amantadine

B. Bromocriptine

C Trihexyphenidyl✔

D.Levodopa (L-DOPA)
Before drug be approved we see the cost of safty and non safty which type of cost:-

A.Cost of illness

B.Cost of benefits

C. Cost of effectiveness✔

D.Cost of utility

:‫للتوضيح‬

.‫ بس عشان فيه مقارنه ما نختارها‬، Safety ‫ شاف ال‬،‫ ما شاف فعاليه‬A: ‫ال‬

‫ تدفع فلوس تأخذ فايده خدمه زياده طبيه اي شي طبي‬B : ‫ال‬

.‫ او ال فنختارها‬safe ‫ بس دام فيه مقارنه يشوف هل هو‬،‫ مقارنه بين دوائين وتاخذ االكثر فعاليه ما يهم حتى لو كان اغلى‬C :‫ال‬

‫ كيف نعرف؟ اذا تحسنت حالة المريض بامراض الكرونك‬،‫ يستخدمون للكرونك‬D: ‫ال‬

renal toxicity)‫يعمل‬Colistin will affect? (

Liver

Kidney ✔

Women newly get pregnant and preform diabetic test in the

beginning of her pregnancy negative, what is your recommendation

regarding gestational diabetic test?

A)No need for testing

B) Do another test between 4 and 12 weeks

C) Do another test between week 20 and 24✔

: ‫تأثيره على ايش‬UGT1A1*28

a--increase irinotecan metabolism✔

‫الباقي كانت مادة ثانية‬b-- decrease irinotecan metabolism

-How is (first) one to prevent medication error?

Pharmacist

Nurse
Doctor✔

Patient

♀‫ ولكن بعد استفسار من دكتور شاف انه الدكتور نفسه‬،‫كنت اشوف انه الصيدلي اول من يمنع‬

-How is (Last) one to prevent medication error?

Pharmacist

Nurse

Doctor

Patient✔

⁃Where to search for off label drug use :

usp drug information ✔

- hospital formulary

‫ هذا ايش اهميته او ليش يعملوا ؟‬skin fold test ‫سؤال‬-

Pulging vasculature

Subcutaneous fat ✔
Third part:

Interaction between warfarin and aspirin?

Protein binding displacement✔

Pt with hypothyroidism taking levothyroxine try to conceive and asked the pharmacist about this drug ?

Continue levothyroxine ✔

Stop levothyroxine

‫ تختاروها‬continue with higher dose ‫لو فيه‬

Elderly with diabetes and another conditions has neuropathic pain?

Gabapentin ✔

Which class from the following can cause diabetes?

Statin✔

ACEIs

Statin taking in bedtime?

Simvastatin✔

Atorvastatin

Least statin has drug-drug interaction?

Simvastatin

Atorvastatin✔

‫ اختاروه النه اقل تفاعل خاصه مع الجريب فروت‬PPFR ‫لكن لو فيه اي واحد من مجموعة‬

Patient with depression and loss of appetite?

Mirtazapine✔

Bupropion
Chlorpromazine MOA?

Block of dopamine 2 receptor✔

Haloperidol side effect?

Extrapyramidal side effect✔

Mood stabilizer drugs for bipolar disorder?

Lamotrigine

Phenytoin

Lithium ✔

Mycophenolate stop before pregnancy?

Four weeks

Six weeks

‫ اسبوع للذكر‬١٢‫ اسابيع لألنثى و‬٦‫المفروض‬

Contraindication in pregnancy?

‫وكثير غيره‬Finasteride ✔

Class B COPD treatment?

Albuterol

Fluticasone

Ipratropium (or tiotropium )✔

Anticoagulant that used just before delivery?

Unfractional heparin✔

Enoxaparin

Wafarin
Hygiene to lower risk of UTI in women?

Urination after sexual intercourse ✔

Infant with psoriasis?

Topical hydrocortisone ✔

Oral hydrocortisone

Rifampicin counseling points ?

‫ وياخذ الدوا على معده فاضيه‬check for liver ‫اللي اعرفه يعمل‬

Zolpidem dose for women?

2.5

5✔

15

20

10 ‫لو قال للذكر نختار‬


1.75 :‫ اما االنثى‬، 3.5 :‫لو قال اقل جرعه بتكون للذكر‬

Patient taking codeine and he is cyp2d6 poor metabolizer?

-Codeine overdose side effect

-Insufficient relief of pain ✔‫كلهم صح بس االصح‬

Virus cells contain?

-Plasmid

-Peptidoglycan

-nucleic acid genome and protein capsid that cover the genome and lipid envelope✔

A Doctor noticed a new side effect for a drug, what study should he wrote?
Case control ✔

clinical trial

Hypersensitivity reaction type ll ?

IgG✔

IgM

IgE

IgA

‫ لو فيه‬IgG ‫ و‬IgM ‫مع بعض اختاروهم‬

First antihypertensive for adult:

Aliskiren

Clonidine

Doxazocin

Amlodipine✔

Patient has acute migraine came to the emergency with a severe migraine headache what medication
will be used?

Sumatriptan✔

Considered first line therapy for the management of partial seizures?

Carbamazepine ✔

Phenobarbital

Gabapentin

Primidone

Pigeon dropping ?

(Cryptococcosis), ✔

Fungus called: cryptococcus


Status epilepticus treatment second line if BDZ didn’t work ?

1.Fosphenytion ✔

2.Levetiracetam

Ethoxamide‫ مش موجود نختار‬bdz ‫لو‬

-patient Hyperthyroidism & antiarrhythmic

A. amiodarone✔

B. dronedarone

-The first one should be informed if medical error?

Safety gp

manager✔

the pharmacist offer handshake to the director,type of communication?

1-verbal

2-non verbal

3-physical✔

85)Which of the following is the most appropriate time to draw a plasma blood

86/sample for digoxin monitoring?

A) 2 hours post-dose

B)4 hours post-dose

C)5 hours post-dose

D)6 hours post-dos✔

/Which of the following tests should be

performed prior to infliximab?


A) HIV screening

B) Influenza screening

C) Hepatitis C screening

D)Tuberculosis screening✔

Which of the following conditions is treated with desmopressin?

A. psoriasis

B. endometriosis

C. nocturnal enuresis✔

D. attention deficit disorder

) A 68-year-old woman with a history of type-2 diabetes and dyslipidemia is

on atenolol and lisinopril. The doctor wants to start statin as her estimated 10-

year ASCVD risk is 10%

Which of the following would be the recommended station in (mg)?

A. Atoravastatin 10mg

B.Atoravastatin 40 mg✔

C. Simvastatin 10 mg

D. Simvastatin 80 mg

Which diuretic safe pregnant?

Thiazide✔

Antihypertensive produce tachycardia?

Hydralazine✔

Reserpine

Methyldopa

Atenolol
How to write enoxaparin prescription ?

a) clexane 80 mg subQ every 12hr

b) enoxaparein 80.0 subQ BID

c) enoxaparin 80.0 subcatanous BID

d) enoxaparin 80 mg subcatanous every 12 hr✔

119)Monitoring of Bactrim (sulfamethoxazole+trimethoprim) ➡ cbc

Because side effect are:

Hyperkalemia✔

decrease WBCs

decrease platelet count

-[Odds ratio in?

a) Case control ✔

b) cohort

-Relative risk? cohort✔

: which one for treatment Migraine ?

a) Dihydroergotamine ✔

b)Alpha ergometrine

Tools used to exam macular degeneration and retinal tear?

A. Snellen chart

B. Retinoscope ✔

C. Ishihara chart

* Doxorubicin mechanism?

A. Alkylating
B. Antimetabolite

C. Metalating

D. ANTHRACYCLINES✔

Which beta blocker less lipid soluble and less CNS side effect ?

- atenolol ✔

- Propranolol

- metoprolol succinate

Drug used in operation surgery?

A. Enoxaparin

B. warfarin

C. Dabigatran

D. Unfractionated heparin✔

Vit C used to decrease?

a/viral infection severity

B/Viral infection duration✔

Natural plant for increase physical activity and reduce stress?

A/chamomile

B/ ginseng✔ ‫اذا جابهم سوا نختار االصح‬

increase physical activity: is ginseng

reduce stress : is chamomile

Drug can cross placenta

A/insulin

B/heparin

C/ glyp✔
‫ لو فيه‬Amoxicillin ‫نختاره‬

All patient with UTI what is the best treatment

A/cephalexin

B/nitrofurantoin

C/TMP/SMX✔

patient with Crohn's disease has been on the maximum dose of budesonide but still uncontrolled,
presented to the ER with acute symptoms what to give?

A- IV cyclosporin

B- IV methylpred ✔

C- Salazine

Class of betrixaban?

A/antibiotic

B/corticosteroid

C/Antiplatelet✔

88-Which of the following is an indication for the use of indapamide?

A. Angina pectoris

B. Cardiac dysrhythmias

C. Essential hypertension✔

D. Acute myocardial infarction

0) Acute ischemic stroke treated with ?

a- Amlodipine

b– hydralazine✔

Pt with G6pD and has UTI?


A, meropenem✔

B, moxifloxacin

C, azithromycin

Which need to calculate CHAD VASC to calculate statin dose

A, pt with stroke ✔

B, pt with IM

C, pt with DM

To Prophylaxis from stroke Use:

Fibrinolytic

Anti platelets ✔

Anticoagulant

Eye drops can used after opening?

10 days

12 days

28 days✔

‫ نختاره‬days 21 ‫لو فيه‬

2-A post menopause old woman is suffering from facial flushing and vaginal drying.

She has done hysterectomy procedure. Which drug of the following ?

should she use?

A-estrogen ✔

B- Progesterone

Pt has Crohn's disease he takes Sulfasalazine

But the symptoms gets worse what is the best option to ?

A-Cyclosporin
B-Acetaminophen

C-Prednisolone✔

Patient with crohn disease he cannot take oral which IV is acceptable ?

A/cyclosporine

B/salazin✔

what is the leads compound?

a-leads to pharmacological activity

b-leads to modification in structure ✔

What is direct agenic category of Isoternin ?

A/ c

B/d

C/x

‫ بس ماكان واضح بالخيار‬Cyp 3A4 ✔ ‫الصح‬

Which of the following is most likely to cause extra para middle effect

Mesoridazine

Thioridazine

Haloperidol✔

3. HIV asymptomatic and CD 4= 500 ask about vaccination ? Hep A/ infulenza /

no vacciene / penomococcal✔

Patient come with acute pain from osteoarthritis which drug not used for this

patient

A. aspirin

B. ibuprofen
C. cortisone ✔

D. acetaminophen

Which of the proteins below is a protease inhibitor controlling blood clotting?

a) Thrombin

b) Plasmin

c) Antithrombin✔

d) Tissue plasminogen activator

Which of the following determinants of health is a 'downstream' determinant?

a) Housing

b) Employment conditions

c) Age

d) Education✔

Patient has accident and take anti psychotic with Cimetidine for heart burn and have panic attack s.e
happened ?

A. cimetidine increase metabolism of drug

B. decrease metabolism✔

What is the change in pedia relative to adult ?

-high volume of distribution , long half life✔

-low volume of distribution , short half life

-high volume of distribution , short half life

-low volume of distribution, long half life

Which the following hormone suppress due to take oral contraception:

A) FSH

B) LH
C) GnRH✔

Which of the following parameters is the most appropriate for PQ's self-evaluation of the effectiveness
of levothyroxine therapy?

a. Increased energy✔

b- Weight loss

c- Improved vision

d- less frequent angina

Which of the following conditions associated with resting membrane potential :

A- Potassium and chloride outside the membrane and sodium inside

B- Potassium and sodium outside, chloride inside

C- Sodium and chloride outside, potassium inside✔

D- Sodium inside, chlorine and potassium outside

which of the following decreases the effect of phenytoin?

a. alcohol✔

b. chronic renal toxicity

c. mild renal impairment

Q- ISO 5 IS which ?

Anteroom, laminar flow✔ , buffer area.

Q- ISO 7 IS which ?

Anteroom, laminar flow, buffer area✔.

Q- ISO 8 IS which?

Anteroom✔, laminar flow , buffer area.

A patient (she was an elderly I think 75) has a seizure, a history of arrhythmia and

is diagnosed with depression. What's the best management ?


A- Bupropion

B- Paroxetine ✔

C- imipramine

‫ مرض‬approved to stop ‫دواء‬

‫ولكن مايعالجه اي من التالي صحيح بنسبة للمرض؟‬


incidence decrease

prevalence decrease

prevalence increase✔

incidence increases

Which of the following anti-arrhythmic drugs can is considered a rate control drug (choices: satalol,
flecainide, amiodarone✔, propofenone)

Medication Regulations :

MOH

SFDA ✔

which one cause iron deficiency

Magnesium hydroxide

B- Levothyroxine

C- Metformin ✔

Laxative slow onset of action

minimum age limit for giving levocetrezine?

a. after 6 months✔

b. after 1 year

c. after 4 yrs
Virus infection increase in?

Lymphocyte✔

Neutrophil

Basophil

C ‫ نختار‬decrease ‫واذا قال‬

Which WBCs responsible for fighting virus infection?

A. lymphocytes ✅

B. Basophils

C. Neutrophils

( note : Neutrophile: bacteria / Lymphocytes: viral / Monocyte: viral/fungi / Esophile: parasite and
inflammatory / Basophile: mediate immunity (histamine))

A 75 yr old women suffering from night bed wetting. Which is the best treatment?

a. darifenacin

b. Oxybutinin ✔

c. Pelvic excercises

d. Tolterodine

A or B and why pls

A 34-year-old man has end stage renal failure and is awaiting a transplant.

Which of the following changes in pharmacokinetics should be considered when

planning drug therapy?

A. Decrease in glomerular filtration rate✔

B. Increase in active tubular secretion

C. Increase in albumin levels

D. Increase in renal metabolism

- Drug enhanced it’s absorption with ranitidine


A- lorazepam

B- Warfarin

C- Aspirin

D- Naproxen✔

Which of the following is not a barrier to community pharmacy's involvement in

the new public health agenda?

a) A propensity for pharmacists to focus on the biomedical model of health

b) The operation of community pharmacy in a retail environment✔

c) A lack of cooperation between pharmacists in the community pharmacy sector

d) The location of many communitie pharmacies

formulary in the hospital this is consider as ?

Remove Watermark Wonder share

PDF element

A. Inter institutional

B. Intra institutional ✔

C. Written communication

D. Non-Verbal communication

function alpha 1-antitrypsin in lungs

A- work as surfactant

B- enhance alveolar exchange✔

Drug cause constipation?

a) Atorvastatin

b) Verapamil✔

c) Nitrate
- A 45-year-old patient with atrial fibrillation is treated with amiodarone 400

mg/day and rivaroxaban 20 mg/day with the evening meal. He has concern about his drugs

routine monitoring.

Which of the following would be the best recommended regarding amiodarone

monitoring?

A. Slit-lamp examination at baseline and annually

B. Renal function tests at baseline and annually

C. Cardiac enzymes at baseline and annually

D. Liver function tests at baseline and every 6 months✔

The authority responsible for the equivalence of the pharmacy certificate coming from

external countries ?

A) Ministry of Education✔

B) Saudi Commission

C) SFDA

D) MOH

A 2 years old child has taken Hepatitis A vaccine and came to take MMR vaccine:

a) Should take MMR vaccine immediately ✔

b) MMR vaccine should be delayed 3 months

c) MMR vaccine should be delayed 6 months

- Antidepressants have teratogenicity effect on pregnant w

A. SNRIs

B. SSRIs

C. MAOIs✔

D. TCAS

Who will sign the form that is sent to SFDA for investigation of a drug ?
A) Physician B) Head nurse

C) Investigator D) Chief pharmacist✔

case about COPD and he asked about the drug that affect the prognosis of the disease:

a) Salmeterol

b) Salbutamol

c) Corticosteroid

d) Ipratropium Br✔

Sever muscle crump , abdominal pain ,nausea , vomiting EXAMINATION reveal hypo adrenal function
immediate ttt?

a) 100mg IV hydrocortisone✔

b) 5mg prednisolone orally

c) 5% dextrose then NS

Hfa for:

Aerosol✔

Capsule

Injection

Q)Which of the following best describe medication adherence:

A)Patient follow instructor of the healthcare provider which written and agreed ✔

B)patient follow recommendation of the HCP regarding timing and dosage

medication

- Ciprofloxacin + metronidazole give to:

A- Mild Crohn's disease

B- Moderate Crohn's disease✔

C- Sever Crohn's disease


D- Not use

Septic shock patient unresponsive to fluid and

CKD what will you Give

A-vasopressin

B-Dopamine✔‫الن عنده مشكله بالكلى‬

C-Norepinephrine

G protein coupled receptor second messenger?

A. AP3

B. CGMP

C. CAMP ✔

A 37-year-old mother breastfeeding her child has developed consistent high blood pressure. The
physician consults the pharmacy to recommend an antihypertensive to control her blood pressure

Which of the following anti-hypertensive is most hazardous for the child?

A. Atenolol✔

B. Verapamil

C. Metoprolol

D. Propranolol

Avoidance of consuming tyramine containing food is recommended with which of the following
medications?

Warfarin

Linezolid✔

Prednisone

Cyclosporine

*Supplement potentiate antidiabetic drugs?


Aloe vera ✔

Chamomile

Echinacea

*Herbal product can potentiate the effect of oral antidiabetic?

Garlic

Ginseng ✔

Green tea

Echinacea

‫ نختارها‬aloe vera ‫اذا مو موجوده وكان فيه‬

Which of the following increase INR?

Garlic✔

Green tea

St. jonnswort

-Newly graduated pharmacist hired in multinational. what would be his access of care?

A. Loses access to public

B. Doesn’t lose access to public

C. Can get healthcare from certain private hospitals✔

-Health coverage for pharmacist employed in multinational drug company:

A.Loses access to public

B. Doesn't lose access to public

C.Can get healthcare from certain private hospitals✔

-Which of the following situation demonstrates a direct association pharmaco epidemiological study?

A) The study reveals that it is not a false association

B) The regression provides misleading statistical evidence


C) The association between two variables is a result of another✔

D) The association between the two attributes is not through attributes

hat is the most common side effects of L-dopa that limit its adherence?

1) depression✔

2)Nausea and vomiting 3) dyskinesia 4)orthostatic hypotension

-Maximum dispensing for prescription of phenobarbital from outpatient department

30✔ 60 90

-Maximum dispensing for prescription of amphetamine from outpatient department

30✔ 90 180

Benzodiazepines ،barbiturate ‫وكذلك‬

Vancomycin trough level?

5-10

10-20 ✔

30-40

Target vancomycin trough is sever infection?

5-10

15-25

15-20✔

Hypoglycemia side effect?

Glyburide ✔

Metformin

pioglitazone

contraindication in ACEI ?
bilateral renal artery stenosis✔

Particle size affect which of the following:

a) Uniformity

b) Homogeneity

c) Dissolution✔

Ramelteon MOA?

Melatonin agonist✔

Melatonin antagonist

-What is the principle of marketing option are?

Gain customer

Satisfaction✔

Increase sales

-Main purpose of marketing?

Sell cosmetic products✔

Gall bladder contraction by which hormone?

cholecystokinin✔

Hormone mainly excreted from kidney ‫؟‬

a- hematopoietic b-erythropoietin✔

‫كلهم صحيحه ممكن فيه لخبطه بنقل الخيارات‬

DM1 patient, give what?

Sulfonylurea

GLP1 (liraglutide)✔
DDP

Pioglitazone

Itraconazole capsule require acidic pH optimal dissolution and absorption. Which is appropriate advice
to optimize the oral absorption of drug?

-Take it on empty stomach

-Take it with full glass of water

-Take it with food ✔

-Take it with proton pump inhibitor

what is the conc w/w if 20 ml of water is required to dissolve 2g substance x?

9.09%✔

10.23%

8.60%

5.7

*Pt take spironolactone and diclofenac what is the result of drug drug interaction ?

Hypokalemia, hypernatremia

Hyperkalemia, hyponatremia✔

Hyperuricemia, hypercalcemia

electrolytes will decrease or increase

‫ ؟‬wheezing cough ‫سؤال حق االزما‬-

B1 agonist

✔ B2

‫ فالوجه وش يستعمل ؟‬acne ‫سؤال مريض طلعتله‬-

A- soap✔

B- lotion
Expect the phenytoin level foe pt with low albumin?

High due to hypo albumin ✔

Low duo to hypo albumin

Drug should decrease when* creatinine increase

- propranolol ✔

- amlodipine

- Levetiracetam

*Which of the following is the benefit from adding of -polyethylene glycol (PEG) in some medications
like peginterferon ?

To extend the half-life of medications✔

To extend the shelf-life of medications


To exchange the effect of the medications

To exchange the elimination of the medications

*Polyethylene glycol in PEG- interferon?

A Increase effect

B Increase solubility✔

●Bivalirudin monitor ?

( inr - aptt✔ - factor xa-..)

●Enoxaparin patient monitor ?

aptt‫ (ماكانت موجوده‬inr - factor x ✔)

●Surgent want to know about DVT in which text book and ask a pharmacist ( DVT american 2013 update
book - Prevention of DVT for orthopedic book - prevention of DVT for non orthopedic book ✔- book of
parentral treatment )
Monophasic contraceptive ?

A) pills deliver the same amount of estrogen and progestin each day for 21 days.✔

B) deliver the same amount of estrogen each day, but the level of progestin is increased about halfway
through the pill cycle.

C) have 3 different doses of progestin and estrogen that change approximately every 7 days.

1)biphasic contraceptive ?

A) pills deliver the same amount of estrogen and progestin each day for 21 days.

B) deliver the same amount of estrogen each day, but the level of progestin is increased about halfway
through the pill cycle.✔

C) have 3 different doses of progestin and estrogen that change approximately every 7 days.

2) triphasic contraceptive?

A) pills deliver the same amount of estrogen and progestin each day for 21 days.

B) deliver the same amount of estrogen each day, but the level of progestin is increased about halfway
through the pill cycle.

C) have 3 different doses of progestin and estrogen that change approximately every 7 days. ✔

Which of the following is the most appropriate advice to give to asthmatic patients as
non-pharmacological management?

A. Air purifiers will help ease the patients’ symptoms

B. OB Weight-loss is unlikely to affect their symptoms now

C. Patients should be trained on breathing exercise programs✔

D. OD. increase physical activity

Who sees pain four hours after eating ?

A) gastric

B) duodenal ulcer✔

Fixation
IgG or IgM✔

which of the following systems can be integrated with computerized?

physician order entry to guide physician during prescribing medications?

A) Smart plumbs

B) B. barcode systems

C) Electronic distribution cabinets

D) Clinical decision support system✔

Which of the following describe glucose uptake and usage by as compared

to other tissues of the body?

A) Brain cell can uptake and use glucose only in the presence

B) Brain cells can uptake and use glucose without being stimul insulin✔

C)Brain cells can uptake and use glucose when glucose is above 100 mg/dL in

the blood

D) Brain cells can uptake and use glucose when the insulin is at high level in the

Body

- Which of the following statements provides the best describtic incremental cost-effectiveness ratio?

A- The cost per extra benefit of a new strategy, independent treatment alternatives

B- The extra cost to obtain an extra benefit when switching alternative to another✔

C- The cost per quality-adjusted life year gained

D- A summary measurement of efficiency

Inflammatory acne need cleansing:

Hydrocortisone 1%

Facial scrub

Mild soap clean✔

Hydroquinone
virus structure contain:

Cell membrane

Cell wall

Plasmid

Nucleic acid genome✔

Mycophenolate stops before pregnancy?

Four weeks

Six weeks✔

The most reliable study to make a clinical decision?

Systematic review ✔

Case control

Cohort study

Which of the following is the interpretation of a P-value of 0.05?

A) There is 0.05% probability that the result is due to random chance

B) There is 5% probability that the results are due to random✔

C) There is 50% probability that the results are due to random

D) There is 95% probability that the results are due to random

Rotavirus contraindication ?

Latex✔

* Mmr CI with egg allergy*

* IPV CI with neomycin allergy*

which is a base or conjugated group?

phenolate✔
tertiary amine

quaternary amine

52- which need to calculate CHAD VASE, to calculate statin dose?

- Patient with stroke

- PT with MI

- Pt with DM

- pt with AFIB✔

what is the most common side effects of

L-dopa that limit its adherence?

1) depression

2)Nausea and vomiting ✔

3)dyskinesia

4)orthostatic hypotension‫ه‬

Which of the following is volatile liquid?

1) Nicotine✔

2) Nitrogen

-Characteristic of iron deficiency ?

anemia

Glossitis

Angular stomatitis

Koilonychia✔

Patient with prolonged qt syndrome should avoid ?

Methadone ✔

Amlodipine
Minocycline

Linezolid

-Drug used to trat hypertension and prophylaxis for headache?

A. Hydralazine

B. phenytoin✔

C. Dapson

D. minoxidil

-Live vaccine and antibody administration?

‫الهواري‬.‫د‬A. Same day different sites ✔✔

B.. Separate 2weeks

‫محمد سليمان‬.‫د‬C. Administer live vaccine then after 1 week antibody✔

D. Separate one month in different sites

‫للتوضيح‬:

‫ سبب اختيار‬C : [ If these live vaccines are administered first, it is necessary to wait at least 2 weeks before
administering the antibody].

.‫*يعطيها بنفس الوقت او يفصلها بدون اهمية للوقت يوم يومين شهر‬inactive+inactive:

‫برضو نفس الشي‬Live+inactive

‫ يوم‬٢٨ ‫يا يعطيه بنفس اليوم بنفس الوقت او يفصل‬Live+live

‫ شهور‬٣ ‫ يجب ان تفصل اتوقع‬immunoglobulin ‫اللي هي‬Live+Antibody

-Patient with daily cough and chest tightness, with 3 – 4 night attacks per week, with limitations in his
daily activity, what is the classification of this asthma type?

A.Intermittent

B.Mild

C.Moderate persistent ✔

D.Sever persistent
-Which of the following large lymphoid organ?

A- liver

B-spleen✔

C- muscle

D- intestine

Medication is an anti-androgen?

Danazol

Tamoxifen

Finasteride ✔

Mifepristone

A‫ يعني‬androgen ‫ويقولو اذا‬

Doxorubicin mechanism?

Alkylating

Antimetabolite

Metalating

Anthracyclines ✔

‫تقريبًا السؤال كذا‬The patient didn't take the third dose of Heb B for long period?

‫اتوقع ذي‬A) give the dose ✔

B) give the additional dose after the third dose

C)repeat from the start

D) make a Test For Heb B to check if he needs to start again


fourth part:

:‫الجزئية ذي تأكدوا من أجوبتها‬

1/ Case pt not need to dely live vaccine

-Pt recently done from chemo

‫اتوقع‬-Pt ended course of steroids ✔

2/ Type II glass containers are?

A.Most inert Glasses and shows high Hydrolytic Resistance✔

B.Suitable for most acidic and neutral aqueous preparations

C.Suitable for Alkaline solutions

D.Suitable for non-aqueous preparations

3/-Largest organ in the human body?

Brain

Liver

Skin✔

4/ Which one of the following statements accurately explain the idea of

‟Diffusion of innovation therapy for health promotion?

A)Psychological process of creativity

B)The way how people develop new idea

C)How new ideas are controlled by a culture

D)The method of disseminating innovative ideas a through a culture✔

5/ Narcotic IV still effective till ?

‫اتوقع ذي األصح‬A. 12 Hours✔✔


B. 24 Hours✔

C. 36 Hours

D. 72 Hours

♀‫ ساعه‬٢٤ ‫ نختار‬mixtur ‫ ولو كتب فالسؤال‬،‫ ساعه اذا كذا‬١٢ ‫يقولو‬

6/ What is the Phenothiazine mechanism of action?

A.Dopamine Antagonist✔

b.Dopamine Agonist

C.Sertonin Reuptake inhibit

7/-Which of the following is the interpretation of the resut of astatistical test denoted P?

A)the null hypothesis HO is rejected if P> 0.05

B) the null hypothesis HO is accepted if P < 0.05

C) the null hypothesis H0 is rejected if p <0.05✔

D)the null hypothesis HO is accepted if p> 0.05

8/-Which of the following is the interpretation of a P-value of 0.05?

A) There is 0.05% probability that the result are due to random chance

B) There is 5% probability that the results are due to random✔

C) There is 50% probability that the results are due to random

D) There is 95% probability that the results are due to random

9/ What is the effect of smoking on theophylline?

A.Inhibition of CYP3A4

B. Inducer of CYP1A2 ✔

C. No effect

D.Decrease excretion of theophylline

10/-Maximum day supply a pharmacist can dispense for diazepam prescription?


‫‪7 days‬‬

‫✔‪30 days‬‬

‫* ‪* Dispensed: 30 / Valid: 7‬للتوضيح‬

‫صالحية الوصفه ‪ ٧‬ايام‪ ،‬لكن مدة الصرف شهر‬

‫نبتة تستخدم لعالج ‪11/‬‬


‫?‪upper respiratory tract infection‬‬

‫✔ ‪-echinchae‬اللي تعالج السعال اتوقع‬

‫‪12/‬‬

‫✔‪-Baglet test ——> Digitals‬‬

‫✔‪-Dry Cough —> Thyme‬‬

‫✔‪-Dragendorf test: Alkaloids‬‬

‫✔ ‪-PH of stomach: 1.5 to 3.5‬‬

‫‪13/ Fingolimod‬‬

‫ايش ‪ test‬اللي المفروض نسويه قبل ما نصرفه؟‬

‫✔‪Echocardiograph‬اتوقع‬

‫‪Sensitivity test‬‬

‫‪ 14-‬سؤال مريض عنده انه بواسير او اشياء بس يستخدم ‪ lidocaine‬وجاء له تحسس منه وش نختار؟‬

‫✔ ‪Pramoxine‬‬

‫‪15-‬سؤال اللي مريض ينقل دواء من مكان معقم إلى مكان معقم باداة معقمه وسأل عن الريسك؟‬

‫‪✔low risk level‬‬

‫‪16-‬سؤال مين يسبب ويولد نبضات القلب؟‬

‫‪✔SA node‬‬
AV node

‫ وسأل ايش آلية عمله؟‬prednisone ‫سؤال‬17-

anti-inflammatory✔

18-‫سؤال مين اللي يحول‬

?pepsinogen to pepsin

-✔Hydrochloric acid (HCl)

*Gastric acid = HCL*

‫ لقياس الغدة الدرقية‬gold standerd ‫سؤال‬19-

TSH✔

‫ برضو تعتبر صحيحه‬COPD ‫ولو فيه‬

OSTEOPOROSIS AND HEART BURN ‫ شخص عنده‬20-

Calcium carbonate✔

‫ ؟‬Starch ‫ سؤال مين اللي يكسر ال‬21-

Amylase✔

‫ ايش نعالجها؟‬red man syndrome ‫ سؤال تبع الفانكومايسين وقال انه يسبب‬22/-

slow rate of vancomycin infusions✔

‫ يتعامل مع ؟‬/‫ فقط وانه هذا ايش يخص‬relatives ‫ سؤال انه شخص يتعالج مع حقته‬23/-

justice✔

‫ ؟‬agranulocytosus ‫ سؤال من اللي يسبب‬24/-

PTU✔
‫‪ 25/‬سؤال ايش قياس‪ Hypothyroidism‬؟‬

‫✔ ‪elevated tsh and decreased t4‬‬

‫‪ 26/-‬مريض عنده اعراض كثيره ومن ضمنها نقص ‪ cerebral blood flow‬على ما اذكر وايش النبته اللي تعالجه؟‬

‫✔‪ginkgo‬‬

‫‪ 27/‬اجاء ال يش تستخدم!‪saw palmetto‬‬

‫✔‪PBH‬‬

‫‪ 28/-‬سوال مين الفئة المفروض ما نعطيها ‪ ACEI‬زي كذا تقريبًا وكان في أربعة خيارات من ضمنهم‬

‫✔ ‪Bilateral artery setenosis‬‬

‫‪29/‬سوال عالج مره تاخذ ‪ zidovidone‬المفروض لمده كم‬

‫‪weeks✔4-6‬‬

‫‪30/-‬سؤال طفل صغير وعنده ‪ otitis media‬؟‬

‫✔‪high dose of amoxicillin‬‬

‫‪Azithromycin‬‬

‫‪31/-‬سؤال مريض بياخد اقراص فينيتوين ‪ 100‬ملجم ‪ 3‬مرات يوميا والدكتور هيحوله لشراب تركيزه ‪10‬ملجم‪ 5/‬مل‪ ،‬هياخد اد ايه من‬
‫الشرب يوميا؟‬
‫ملجم ‪400‬‬

‫ملجم ‪200‬‬

‫✔ملجم ‪300‬‬

‫ملجم ‪500‬‬

‫‪32/-‬سؤال الطفل اللي اخذ ‪ heb a immunoglobin‬وجاي ياخد‪ MMR‬؟‬

‫نعطيه مباشرة✔‬

‫ينتظر ‪ 3‬اشهر‬
‫‪6‬اشهر‬

‫‪9‬اشهر‬

‫للتوضيح‪ :‬الزم بنفس اليوم بمكانين مختلفين بالجسم واذا تأخر يوم يخلي فاصل ‪ ٢٨‬يوم‬

‫‪33/-‬سؤال ‪ promethazine‬هو يستخدم استخدام‬

‫✔‪antihistaminic‬‬

‫‪antipsychtic‬‬

‫‪34/‬سؤال اي واحد يتاخذ مع الكانسر اللي يكون صلب‬

‫?‪Used for solid tumers‬‬

‫✔‪ Dostarlimab ✔ Entrectinib✔ pembrolizumab‬اللي جا‬

‫كلهم صح‬

‫؟‪ hyperkalemia‬سؤال جالي واحد‪35/‬‬

‫‪ca gluconate‬‬

‫‪Na carbonate‬‬

‫✔‪Ca resin‬‬

‫‪36/‬سؤال ال ‪ vaccine‬للحامل تاخده مع كل حمل غير‪ influenza‬؟‬

‫✔ ‪- Tdap‬‬

‫‪37/-‬سؤال ممرضة تعرضت لدم مريض ‪ hep b‬ومش متطعمة؟‬

‫✔ ‪- ig + vaccination‬‬

‫‪38/‬سؤال الرجل اللي ياخد ‪ rivastigmine‬وجاله ‪ nausea‬ياخذ ايش؟‬

‫✔‪donepezil‬‬

‫‪Galantamine‬‬

‫‪Memantine‬‬

‫‪Tacrine‬‬
‫‪ 39/‬سؤال اي العالجات‪ Mast cell activator‬؟‬

‫✔ ‪-platelet activating factor‬‬

‫؟ )‪ KOH (potassium hydroxide‬سؤال ال‪40/‬‬

‫✔ ‪Lead‬‬

‫‪41/-‬سؤال اعراض شخص عنده ‪ angina‬وبيتعب وقت الحمام ولوما يمشط شعره ولوما ينام يحط مخدتين تحته ويتعب حتى وقت‬
‫الراحه يمكن يجيه ‪attack‬‬

‫؟ ‪angina‬‬

‫‪Class i‬‬

‫‪Class ii‬‬

‫‪Class iii‬‬

‫✔ ‪Class iv‬‬

‫‪42/-‬جاب عالج ‪ anticancer‬من عائلة‪alkylating:‬‬

‫✔‪cyclophosphamide‬‬

‫‪43/‬جاب ‪ case‬وحده عندها ضغط ومشاكل أخرى‬

‫عمرها ‪ 61‬حاولت تعمل‪life style modifications‬‬

‫منذ اسبوعين وما نفعها ضغطها ايش انسب ‪ management‬لها؟‬

‫‪thiazide‬‬

‫✔ ‪Amlodipine‬النه افضل شي لكبار السن‬

‫‪44/-‬سؤال اعراض واختناق وأشياء كثيرة وجاب خيارات من ضمنها؟‬

‫‪Pneumonia‬‬

‫✔‪COPD‬‬
‫‪45/-‬جاب صوره من مرجع فيها كالم شرح عالجات ‪ copd‬وسأل ايش عالج ال‪ ، copd‬طبعا في الصوره كاتب عالج ‪copd:‬‬

‫✔ ‪-Daily inhaled corticosteroids and saba as need‬ممكن ذي‬ ‫♀‬

‫‪-Daily/ inhaled corticosteroids and laba‬تقريبًا‬

‫?‪-Which of the following acts as beta blocker and partial agonist‬‬

‫✔‪pindolol‬‬

‫اتوقع الخيارات كذا تقريبًا أو ‪:‬‬

‫‪1....‬مستبعد‬

‫‪2....‬مستبعد‬

‫✔‪3-inhaled corticosteroids and saba as need‬هل صحيحه؟‬

‫‪4-Saba daily and corticosteroids as needed‬‬

‫‪46/‬سؤال ال ‪ epinephrine‬لوما يحصل له تفاعل ويتحول إلى ‪ phenylephrine‬ايش مفعوله بيكون؟‬

‫✔‪increase alpha 1 selectivity‬‬

‫‪47/-‬سؤال كيف تساعدي شخص ينام بشكل كويس ويضبط نومه وفي خيارات من ضمنها ‪:‬‬

‫‪-‬يأكل شوكالته قبل ما يروح ينام‬

‫‪-‬او يشرب مويه كثير قبل ما يروح ينام هو تمام بس يشرب ماء زياده عن اللزوم ما اخترته‬

‫‪-‬الخيار الثالث قال يروح ينام ويستيقظ بنفس الميعاد ✔‬

‫‪48/‬سأل ال ‪phase 3‬بتركز على ايش؟‬

‫‪Efficacy & and therapeutic range‬‬

‫✔‪Efficacy. safety and therapeutic range‬‬

‫‪49/‬جايب ايه اللي بيقلل تأثير الوارفارين و االختيارات‬

‫‪St John warts‬‬

‫✔ ‪Rifampcin‬االثنين صح بس بيقولو هذا أقوى‬


‫ حق العالجات؟‬Emergency cart cabinet ‫ايش فايده‬50/

Allow timely providing emergency drugs✔

‫ ؟‬skin ‫ايش نوع البكتيريا اللي ع‬51/

staphylococcus epidermis✔

‫؟ تسبب التهاب لوز‬/ Bacteria cause Tonsillitis52

Streptococcus pyogenes✔

‫يعتبر جهاز مناعي ثانوي ؟‬/ a secondary immune system?53

‫اللوز‬Tonsillitis ✔

‫ (جلده جاف) ومرض جلدي ايش يعطونه؟‬dry skin ‫واحد عنده‬54/

Cream

Lotion

Ointment ✔

Sloution

‫؟ اكبر عضو‬/The largest organ in the body55

skin ✔

liver

blood

brain

‫ ؟‬the largest lymphoid organ‫اما لو قال‬

spleen ✔

،‫ مريض عنده سكر وضغط وقلب وكل شي وياخذ وارفارين وميتفورمين وقليبزايد وملتيفيتامين تحاليله كلها رافعه معادن وكيرياتين‬56/
‫نوقف أي دواء؟‬

‫ النه ممنوع لمرضى القلب‬Glipizide✔‫االصح‬


2030 ‫االمراض المستهدف‬57/

‫ بيجيب اثنين ويطلب الثالثه‬,)‫(امراض القلب والسكري والسرطان‬

‫ ؟‬pharmacokinetics ‫سؤال عن‬58/

- zero order ✔

- first order

:‫شخص اتسمم بسم فئران وحصل اعراض كذا و كذا مفيش فيها نزيف واالختيارات كان فيها‬59/

Warfarin

Atropine

Aluminium phosphide ✔

: ‫فيه سؤال مشابه له اللي هو‬

-About lady ingested a rodenticide in which lab reports were given. Serum potassium seems to be low.
Which among is possible?

Zinc phosphide

Aluminium phosphide

Aluminium hydroxide

Barium carbonate ✔

‫ ؟‬Saitagliptin‫ايش تقيس لواحد ياخذ‬60/

Live

Creatin clearance✔

‫سايد افكت لدواء جديد يعتمد على ايش؟ تقريبا كذا‬61/

Potency

Effenty

Affectiy
Thrputic range✔

‫ ؟‬aspart dose ‫متى نقيس مستوى السكر بعد األكل عشان نشوف اذا بنعدل‬62/

min15

min30

h1

h✔2

63/

‫وش يعتبر؟‬Aminoglucosid

Time depnded

Conc depede✔

Alkaloid‫الغاز اللي يطلع لما نحط الفوار مع المويا؟‬64/

Co2✔

Oxygen

‫ ؟‬complications ‫ ايش نعطيه عشان نقلل ال‬meningitis ‫مريض عنده‬65/

Cortison ✔

‫ ؟‬evidence ‫مين اقوى في ال‬66/

Systemic review✔

RCT

Cross sectional

Cohort

‫ ممنوع تاخذ ايه؟‬arrhythmia ‫وبتعاني من‬..‫ وأدوية تانيه مش فاكرها‬metoprolol ‫مريضه بتاخذ‬67/

Sotalol✔
Amiodaron

Flecainide

PR interval ‫ في‬events ‫و سأل عن ال‬action potential ‫جاب رسمة ال‬68/

- ventricular depolarization

- atrioventricular conduction✔

- atrial depolarization

- Ventricular repolarization

‫ ؟‬interactions ‫ بيعمل‬antihyperlipidemia ‫سؤال عن اكتر‬69/

ezetimibe

Statin ✔

Niacin

‫تقريبًا السؤال‬/ ?Plant used in inflammation 70

Mume fructus✔

71/

-The major cation in the intracellular fluid?

K+✔

-The major cation in the extracellular fluid?

Ca+✔

‫ ؟؟‬enalapril‫ و الدكتور هيغيرله الدواء ياخد الدواء التانى بعد كام ساعه من ال‬enalapril ‫لو واحد ماشي على ال‬72/

24h

48h

36h ✔

‫ ؟‬NSAIDs‫ايه اول حاجه تتطمن عليها لو واحد كبير بالسن ماشي على‬73/
‫‪liver‬‬

‫‪Stomach bleeding‬‬

‫✔ ‪Renal‬كلوي‬

‫‪74/‬لو واحد ماشي على ‪ NSAIDs‬فترة طويله و جاله ‪ esophageal erosins‬بعد ماعرف بالمنظار ايه العالج؟‬

‫✔‪Lansoprazole‬‬

‫‪Sacralfate‬‬

‫‪Antodine‬‬

‫‪75/‬واحد بياخد جنتاميسين ماينفعش ياخد معاه ايه؟‬

‫و كان فيه✔ ‪ferosamide‬‬

‫‪76/‬ال ‪ tpn‬باين بنعمله لما يكون العيان مأكلش حاجه من اد ايه؟‬

‫مأكلش اكتر من اسبوع‬


‫عشر ايام‬

‫خمس ايام✔‬

‫‪/ Phenytoin 77‬بيشتغل على ريسبتور اسمه ايه ؟‬

‫‪Na-K-ATPase, the GABAA receptor complex, ionotropic glutamate receptors, calcium channels and sigma‬‬
‫‪binding sites‬‬

‫‪Mast cell‬بتمسك ف ريسيبتور اسمه ايه؟‬

‫✔‪Siglic‬‬

‫ال ‪ Antibody‬بيمسك ف ايه بالبكتيريا؟‬

‫✔ ‪Antigen‬‬

‫‪78/‬كان بيسأل ايه الدواء اللى بيحتاج تعليمات كتير؟‬

‫‪potassium‬‬
Methotrexate ✔

‫حاجات تانيه مش فاكرها‬

‫ يبقا ماينفعش يستخدم دوا ايه؟‬DHFR ‫مشكله في ال‬79/

methotrexte ✔

DHFR ‫ دواء الميثو يشتغل على االنزيم‬:‫للتوضيح‬

DHFR: dihydrofolate reductase‫اختصار‬

80/ Digoxin main effect?

Decrease preload

Decrease afterload

Increase heart contractlity✔

81/ First line h1n1?

oseltamivir✔

82/Test for neutropernia ?

1-ANC ✔

2-WBC

83/ Antimetabolite anticancer ?

Gemcitacbine✔

Cyclophosphamide

84/Which diuretic listed below would have the [greatest] blood pressure lowering effect then given in
‫أي من المدرات لها اكبر او اعظم تأثير في خفض سكر الدم‬patients with an eGFR below 60 ml/min?

Chlorthalidone 25 mg daily

Spironolactone 25 mg daily (cI < 20)

Triamterene 25 mg daily
‫االقوى‬Furosemide 40 mg daily ✔

85/How to prevent sexual transmitted disease

Condom✔

86/Urinary clearance is considered the gold standard for measuring GFR?

inulin✔

‫ضررا على المعدة؟‬


ً ‫اكثرهم‬87/Who is the most harmful to the stomach?

Celocoxibe

Ibuprofen

Naproxen ✔

Diclofenac

‫ أول تدخل هام فورا إعطاء؟‬Perforated colon ‫سؤال عن مريض اشتخص‬88/

‫اوميبرازول وريد‬-

‫فانكوميسين‬-

✔‫مترونيدازول او سلفا‬-

‫اختيار رابع ناسيه‬-

sucralfate‫نختار مترونيدازول واذا مو موجود نختار‬

‫مريض معاه ألم من سنتين وماعنده مشاكل ثاني الم بشعور حرق وطعن وماحدد مكانه؟‬89/

)‫ اتوقع (اقل أعراض‬ibuprofen✔

gapapentin

‫ ؟‬Metformin ‫متى تاخذ‬90/

A- After meal 2 hr

‫ اتوقع‬B -Divided the tab during meal✔


‫‪C- Before meal 2hrs‬‬

‫‪D -On empty stomach‬‬

‫واذا حصلنا باالختيارات ‪ XL‬هي االصح حبة قبل النوم‬

‫‪91/‬سؤال ب ‪ cornary arteri disease‬اي انزيم مرتفع من انزيمات القلب؟‬

‫✔‪Troponin i‬‬

‫‪Troponin t‬‬

‫‪92/‬شخص ياخذ ‪ verpamil‬كم راح ياخذ ‪ simvastatin‬؟‬

‫✔ ‪Simva 10 mg‬‬

‫‪Simva 20 mg‬‬

‫‪93/‬ايه الرقم الثابت في ‪ therapeutic index‬عشان اقدر احدد اذا خطر او ال ؟ يعني اذا كان‪ wide or narrow‬؟‬

‫يعني لو ‪ therapeutic index =4‬؟‬

‫✔ ‪narrow‬‬

‫‪wide‬‬

‫*اذا كان الرقم صغير يعتبر‪narrow‬‬

‫واذا كان كبير يعتبر‪wide‬‬

‫الصغير مثال من واحد لين اقل من عشره‬


‫من عشرة وفوق يعتبر*‪wide‬‬

‫‪94/‬سؤال أي من الديوريتك هذول ‪ potasy-sparing diuretic‬؟‬

‫‪furosemide‬‬

‫✔‪Spironolactone‬‬

‫‪95/‬كيس طويلة وفيها صورة الب ريزولت يوم تنزل الخر سطر يسأل وش الدواء الي يسبب كونستبيشن ودارك ستول من االدوية الي‬
‫ياخذها ومن ضمنها حاط فورم من فورمات االيرون‪..‬‬

‫✔ ‪iron‬‬
‫ ؟‬non essential ‫ وايش‬Essential ‫ ايش‬amino acid ‫اسئلة على‬/96

:‫ بجملة‬٩ ‫عددهم‬
I love lucy very much please try to help arginine

Non essential = 11

Total = 20

‫ ؟‬creatinine clearance for child ‫اسم معادلة ال‬/97

✔Schwartrz

‫ للي عندهم‬Vit D ‫وش الفورم المناسب من‬/98

‫ ؟‬kidney impairment

✔Calcitriol 1,25 dihydroxycholecalciferol-

‫استركشر و صيغة االسبرين؟‬/99

✔C₉H₈O₄

‫؟‬Fat soluble vitamins /100

‫ ✔ كلهم صح‬Vitamins A, D, E, K

:vitamin cause blindness ‫ الفيتامين اللي نقصه الشديد بالطفوله يسبب العمى؟‬/101

✔ Vitamin A

‫؟‬Antibiogram used for /102

is an overall profile of antimicrobial susceptibility testing results of a specific microorganism to a battery(


✔)of antimicrobial drugs

‫ مجموعة الفلور تعمل ايش؟‬Ciprofloxacine ‫ استركشر‬/103

Increase lipophilicity and increase(

✔ .)peneteration of the cell wall


‫ يبقى سبب االنيميا ايش؟‬،‫ تحليل وباين فيه انه الهيموجلوبين نازل‬:‫كيس‬/104
Vit B12

Folic acid

‫ومدري ايش بعد‬


: MCV ‫ نحدد حسب ال‬:‫*للتوضيح‬

: ‫ وسببها‬microcytic ‫ بتبقي‬٨٠ ‫*لو اقل من‬

iron deficiency anaemia

: ‫ بيبقي سببها‬macrocytic ‫ بيبقي‬١٠٠ ‫*ولو اكتر من‬

B12 or B9 (folate) deficiency

:‫) يبقى سببها‬١٠٠-٨٠( Normal ‫*لو كان‬

Normcytic anaemia (A B C D)

‫ ؟‬brown : urine ‫ دواء بيعمل ال‬/105

✔ nitrofurantoin

‫؟‬The largest capsule size /106

✔ 000

‫؟‬How to make floor stock accessible for nurses /107

✔ My make it in every floor

:‫ من ضمن الخيارات‬،)Amoebiasis( ‫ عامله تحليل وظاهر بالتحليل أميبا‬:‫كيس‬/108

✔ epecacanhua /epeca

‫تأثير التدخين على الحمل تقريبًا ؟‬/109

✔Decrease blood flow of utrine

‫؟‬Side effect of ginkgo /110


‫‪✔Bleeding‬‬

‫‪/111‬جاب ‪ Natural product or plant‬ممكن تستعمله لعالج السكر‪ ،‬وماكان فيه ‪ ، ginseng‬من ضمن الخيار‪:‬‬

‫‪✔Aloe vera‬‬

‫‪/112‬صيدلي خريج جديد وتعيّن بشركه‪ ،‬كيف يتعالج؟‬


‫مستشفيات حكومية‬

‫مستشفيات خاصة (‪✔ )private hospital‬‬

‫‪/113‬لو نبي نعمل ‪ test‬لـ ‪ liver‬نشوف سليم او غير سليم‪:‬‬

‫‪Creatinine‬‬

‫‪Urine analysis‬‬

‫✔‪Albumin‬‬

‫‪Mechanism of sitagliptin /114‬؟‬

‫‪✔A Dipeptidyl Peptidase IV Inhibitor‬‬

‫‪/115‬واحد عنده ‪ Diabetes type2‬نعمله قياس النظر متى؟ ماكان جايب بعد سنه‪ ،‬من ضمن الخيارات‪:‬‬

‫‪ Infestation or check‬للنظر✔‬ ‫‪every year‬‬

‫يعني كل سنه في نفس اكتشاف المرض‬

‫‪/116‬كيف نعمل ‪ Monitoring of warfarin‬؟‬

‫‪ INR and enoxaparin‬تنفع‬

‫اذا جاب (‪ )gum bleeding‬نختارها على طول واذا ماجاب نختار ‪:‬‬

‫‪✔Xa Factor‬‬

‫*للتوضيح‪ INR :‬تبع ال ‪warfarin‬‬

‫اما ال ‪ APTT‬تبع ال ‪heparin‬‬

‫‪/117‬كيس تحليل ‪ orange red precipitate‬؟ او ‪ orange red prostate‬؟ مو متاكده من السبلنق‪ ،‬الخيار‪:‬‬
✔Alkaloid

:‫ من ضمن الخيارات‬،‫ سريعه‬OTC ‫ وعنده ضغط ويبغى حاجه‬،runny nose ‫واحد عنده‬/118

Paracetamol

✔oxymetazoline

:‫من ضمن خيارات العالج‬..‫ وكحه ناشفه‬control hypertension ‫واحد عنده‬/119

✔ dextromethorphan

:‫ الخيارات‬،supportive fluid ‫ ؟ ماكان فيه‬Antiseptic shock ‫اول حاجه نستخدمها بـ‬/120

✔ Norepinephrine

Dopamine

Dopamine ‫ نختار‬CKD ‫ اذا كان عنده‬:‫للتوضيح‬

‫فيه مرض ظهر بالمنطقه والحاالت الجديده اللي تظهر منه اللي هي؟‬/121

✔ incidence

‫ وانت صيدلي‬Isotretinoin ‫مكتوب لك‬/122

‫؟‬ethics ‫وانت صرفته لها كذا يبقى مخالف ألي‬..‫والبنت مكتوب لها حامل‬

✔ Non-maleficence

‫ ؟ بالترتيب‬Discovery of drug ‫خطوات‬/123

Taeget > Bioassays > Lead compound > Pharmacophore study

‫ من ضمن‬,23 ‫ لقوه‬voncomycin level ‫ ل‬monitoring ‫ لما عملو‬vancomycin ‫ وتاخذ‬infection ‫مريضه وعندها‬/124


: ‫الخيارات‬
Decrease infusion rate

‫ ✔اتوقع‬Decrease dose
‫ بـ ايش؟‬check or montoring ‫ نعمله‬،‫ بياخذ حاجه للكوليسترول‬anti-trylipidemia ‫مريض بياخذ‬/125

✔creatine kinase

:‫ من ضمن الخيارات‬،‫وحده مرضعه ايش اللي ممكن يأثر بهرمون الحليب‬/126


A.Progesterone.

‫✔ اتوقع ذي الصح‬B. Prolactin

: ‫ نشوف السؤال طالب‬:‫*للتوضيح‬


! Hormone ‫ وال‬Contraceptive pills

Progesterone ‫ يبقى‬pills ‫لو‬

Estrogen / androgen ‫لو هرمون يبقى‬

** :‫**بطريقة أخرى‬
Which contraceptive pills prevent lactation ?

progesterone -1

estrogen -2

✔combined -3

‫ وش سببها ؟‬diarrhea ‫ وجته‬،‫ تقريبًا‬magnesium carbonate ‫ و‬proton pump ‫ و‬codiene ‫ اخذ ادويه كثير منها‬:‫كيس‬/127

✔ megnesium

** :‫**معلومة‬
)‫ (امساك‬constipation : ‫ يعمل‬Aluminium

)‫ (اسهال‬diarrhea : ‫ يعمل‬Magnesium*

You might also like